You are on page 1of 67

1.

A 27-year-old woman presents with a 9-month history of bloody diarrhea and crampy
abdominal pain. Three weeks ago, she noticed that her left knee was swollen, red, and painful.
Her temperature is 38°C (101°F), respirations are 32 per minute, and blood pressure is 130/90
mm Hg. Abdominal palpation reveals tenderness over the left lower quadrant. Laboratory
studies show moderate anemia, with a hemoglobin level of 9.3 g/dL. Microscopic examination
of the stool reveals numerous red and white blood cells. A diffusely red, bleeding, friable
colonic mucosa is visualized by colonoscopy. The colon is subsequently removed and the
surgical specimen is shown in the image. Which of the following is the most likely diagnosis?

a) Adenocarcinoma
b) Ulcerative colitis
c) Carcinoid tumor
d) Crohn disease
e) Pseudomembranous colitis

2. A 21-year-old man is brought to the emergency room with symptoms of acute intestinal
obstruction. His temperature is
38°C (101°F), respirations are 25 per minute, and blood pressure is 120/80 mm Hg. Physical
examination reveals a mass in the right lower abdominal quadrant. The patient subsequently
undergoes surgery, and a segmental lesion involving the terminal ileum is resected (shown in the
image). Which of the following is the most likely diagnosis?

a) Adenocarcinoma
b) Carcinoid tumor
c) Crohn disease
d) Pseudomembranous colitis
e) Ulcerative colitis
3. A 25-year-old woman presents with persistent bloody diarrhea of 4 weeks’ duration. She has
experienced severe abdominal cramping for the past 3 days. Her temperature is 38°C (101°F),
respirations are 22 per minute, and blood pressure is 120/70 mm Hg. Physical examination reveals
abdominal tenderness and mild abdominal distension. Bowel sounds are diminished. Laboratory
studies show mild hypochromic, normocytic anemia. Stool cultures are negative for pathogens,
and no ova or parasites are
detected. A blood test for Clostridium difficile toxin is negative. Rectosigmoidoscopy shows
hemorrhagic mucosal lesions in
the distal colorectal region. A biopsy of the colon reveals crypt abscesses, basal
lymphoplasmacytosis and crypt distortion.
Which of the following represents the most common extraintestinal manifestation of the colonic
disorder in this patient?
a) Arthritis
b) Cystitis
c) Gastritis
d) Pancreatitis
e) Sepsis

4. 32 y.o female compliance diarrhea, vomiting, nausea, abdominal pain, weakness, modychange;On
examination: Her blood pressure 120/70mm.hg, RR-25, HR-90/min, the puls is regular; On chest
examination : normal breath and heart sound; on palpation of the abdomen painless and tenderness.
CBC is normal range, blood occult test is negative
What is your suspected diagnosis?
a. Peptic ulcer disease
b. GERD and chronic enteritis
c. Irritable bowel syndrome
d.Ulcerative colitis

5. A 25-year-old woman is brought to the emergency room with symptoms of acute intestinal
obstruction. The patient has an
8-month history of blood-tinged diarrhea and cramping abdominal pain. Her temperature is 38°C
(101°F), and respirations are
32 per minute. There is abdominal tenderness to palpation. Laboratory studies show moderate
anemia, with serum hemoglobin
of 9.3 g/dL. Microscopic examination of the stool reveals numerous RBCs and WBCs. A CT scan
of the abdomen shows massive
distention of the transverse colon. Which of the following is the most likely underlying cause of
this patient’s colonic disorder?
(A) Adenocarcinoma
(B) Carcinoid tumor
(C) Crohn disease
(D) Pseudomembranous colitis
(E) Ulcerative colitis

6.Which of the following is the most common cause of ulcerative colitis-related mortality?
a) Colonic adenocarcinoma
b) Toxic megacolon
c) Perforated colon
d) Colonic infarction
7. Which of the following is the preferred therapy in mild ulcerative colitis confined to the rectum?
a) Topical mesalazine given by suppository
b) Oral prednisone
c) Intravenous azathioprine
d) Hydrocortisone given by enema

8.Which of the following serologic markers is most indicative of ulcerative colitis?


a) Antineutrophil cytoplasmic antibodies (ANCA)

b) Anti-Saccharomyces cerevisiae antibodies (ASCA)

c) Anti-outer membrane of porin C (OmpC)

d) Anti-I2
9. A 46-year-old female presented to the GP with a change in bowel habit for the past 2 months. She is
having looser stools than normal with blood mixed in with the stools. There is no history of any
abdominal pain. What would be your first choice investigation?
a. Abdominal X-ray
a) b. Faecal occult blood
b) c. Abdominal CT
c) d. Ultrasound abdomen
d) e. Colonoscopy

10.In mild ulcerative colitis confined to the rectum, which of the following is the preferred treatment?
a) Enemas
b) Rectal foams
c) Topical mesalamine
d) Systemic steroids

11.A 41- year -old male patient presented with recurrent episodes of bloody diarrhea for 5 years.
Despite regular treatment with adequate doses of sulfasalazine, he has had several exacerbations of his
disease and required several weeks of steroids for the control of flares. What should be the next line of
treatment for him ?
a) Methotrexate
b) Azathioprine
c) Cyclosporine
d) Cyclophosphamide
12.Best treatment of refractory perianal fistula in Crohn's disease:
a) Fistulectomy
b) Infliximab
c) Olasalazine
d) d .Mesalamine

13.Which of the following is the established biological therapy for Crohn's disease?
a) Anti TNF a -antibody
b) IL-1 antagonist
c) IL - 6 antagonist
d) IL-8 antagonist

14.A highly sensitive and specific marker for detecting intestinal inflammation in ulcerative colitis
is?
a) CRP
b) Fecal lactoferrin
c) Fecal calprotectin
d) d . Leukocytosis
15.Chronic inflammatory bowel disease is associated with:
a) Chronic hepatitis
b) Fibrosis
c) Cholangiosarcoma
d) Primary sclerosing cholangitis
16.Skip granulomatous lesions are seen in :
a) Ulcerative colitis
b) Crohn's disease
c) Whipple's disease
d) Reiter's disease
17. Invariably involved site in ulcerative colitis:
a) Sigmoid colon
b) Transverse colon
c) Ileum
d) Rectum
18.Regional enteritis (Crohn’s Disease) is characterised by:
a) Transmural thickening;
b) Diffuse involvement;
c) Exacerbations and remissions;
d) Severe diarrhoea;
19.A 32-year-old woman has a history of chronic diarrhea and gallstones and now has rectovaginal
fistula. Which of the following is the most likely diagnosis?
a) Crohn disease
b) Ulcerative colitis
c) Systemic lupus erythematosus
d) Laxative abuse
20. A 25-year-old man is hospitalized for ulcerative colitis. He has now developed abdominal
distention, fever, and transverse colonic dilation of 7 cm on x-ray. Which of the following is the best
next step?
a) 5-ASA
b) O ralsteroids
c) IV antibiotics and prompt surgical consultation
d) Infliximab
21. A 45-year-old man with a history of ulcerative colitis is admitted to the hospital with 2 to 3
weeks of right upper quadrant abdominal pain, jaundice, and pruritus. He has no fever and a normal
WBC count. Endoscopic retrograde cholangiopancreatography (ERCP) shows multifocal strictures
of both the intrahepatic and extrahepatic bile ducts with intervening segments of normal and dilated
ducts. Which of the following is the most likely diagnosis?
a) Acute suppurative cholangitis
b) Cholangiocarcinoma
c) Primary sclerosing cholangitis (PSC)
d) Choledocholithiasis with resultant biliary strictures
22. A 25-year-old man is hospitalized for ulcerative colitis. He has now developed abdominal
distention, fever, and transverse colonic dilation of 7 cm on x-ray. Which of the following is the
best next step?
a) 5-ASA
b) Oral steroids
c) IV antibiotics and prompt surgical consultation
d) Infliximab
23. A 35-year-old woman has chronic crampy abdominal pain and intermittent constipation and
diarrhea, but no weight loss or gastrointestinal bleeding. Her abdominal pain is usually
relieved with defection. Colonoscopy and upper endoscopy with biopsies are normal, and stool
cultures are negative.
Which of the following is the most likely diagnosis?
a) Infectious colitis
b) Irritable bowel syndrome
c) Crohn disease
d) Ulcerative colitis
24. Loperamideshould be avoided in patients with active UC due to its ability to induce which one
of the following complications?
a) Toxic megacolon
b) Intestinal stricture
c) Fistula formation
d) Pancreatitis

25. Which one of the following disorders is a possible extraintestinal manifestation of inflammatory
bowel disease?
a) Erythema nodosum
b) Migraine headaches
c) Asthma
d) Sinusitis
26. A 58-year-old man presents to the emergency room with a temperature of 102°F, abdominal
pain localizing to the left lower quadrant, and mild rebound tenderness. Which of the following
diagnostic tests is the best next step?
a) Barium enema
b) Flexible sigmoidoscopy
c) CT imaging of the abdomen
d) Laparoscopic examination
27. Which of the following features is not consistent with the diagnosis of irritable bowel
syndrome?
a) Abdominal pain relieved with defecation
b) Sensation of incomplete evacuation
c) Passage of mucus
d) Nocturnal awakening with pain or diarrhea
28. A young girl presents with abdominal pain and a recent change in bowel habit, with passage of
mucus in stool. There is no associated blood in stool and symptoms are increased with stress.
The most likely diagnosis is -
a) Irritable bowel syndrome
b) Ulcerative colitis
c) Crohn’s disease
d) Amebiasis
29. A 40 year old male presents with chronic diarrhea for one year. On investigation, the findings
obtained were crypt abcess, crypt atrophy, cryptitis and mucositis. Mostlikelydiagnosisis-
a) Crohn'sdisease
b) Ulcerativecolitis
c) Microscopiccolitis
d) Collagenouscolitis
30. A patient gives chronic history of diarrhoea and blood in stool presents with multiple fistulae in
the perineum and multiple stricture in small intestine. The diagnosis is -
a) Crohn's disease
b) Radiation enteritis
c) Ulcerative colitis
d) Ischemic bowel disease

31. Which of the following excludes a diagnosis of irritable bowel syndrome:


a) Relieved by defacation
b) Straining during stool passage
c) Passage of blood per rectum
d) Change of stool form
32. Which of the following features are associated with Irritable Bowel Syndrome:
a) Weight loss
b) Anorexia
c) Abdominal distension
d) Blood in stool
33. Which is true of the complications of Crohn’s disease:
a) perianal complications occur in 50 % of patients;
b) gastrointestinal (GI) bleeding is common and often life-threatening;
c) toxic megacolon occurs in 6 % but almost never perforates;
d) there is no increased risk of neoplasm of GI tract unlike ulcerative colitis;
34.A 40-year-old woman presents with a 2-month history of burning epigastric pain that usually occurs
between meals. The pain can be relieved with antacids or food. The patient also reports a recent history
of tarry stools. She denies taking aspirin or NSAIDs. Laboratory studies show a microcytic,
hypochromic anemia (serum hemoglobin = 8.5 g/dL). Gastroscopy reveals a bleeding mucosal defect in
the antrum measuring 1.5 cm in diameter. An endoscopic biopsy shows that the lesion lacks mucosal
lining cells and is composed of amorphous, cellular debris and numerous neutrophils. Which of the
following is the most important factor in the pathogenesis of this patient’s disease?
a) Achlorohydria
b) Acute ischemia
c) Autoimmunity
d) Gastrinoma
e) Helicobacter pylori infection

35.A 45-year-old man presents with long-standing heartburn and dyspepsia. An X-ray fi lm of the chest
shows a retrocardiac,
gas-filled structure. This patient most likely has which of the following conditions?
a) Esophageal varices
b) Esophageal webs
c) Hiatal hernia
d) Mallory-Weiss syndrome

36.Ulcerative colitis is least associated with which of the following?


a) Increased risk of colon cancer
b) Bloody diarrhea
c) Decreased risk of occurrence with smoking
d) Anal fissures
e) Isolated proctitis
37. A 24 years old female college student complains of diffuse lower abdominal pain. It has been
present for approximately 5 months and gets better with defecating. The onset is associated with more
frequent and loose bowel movement ( 3- 5x/day). She denies blood or bloating. No weight loss or night
time symptoms. Whatisthemostlikelydiagnosis?
a) Ulcerativecolitis
b) Irritablebowelsyndrome.
c) Crohn'sdisease.
d) Lactoseintolerance. 
38. For a 25 - year - old female complaining of chronic constipation for 4 months, the first line
treatment is:
a) orallaxatives
b) referto GI specialist
c) change life style and diet
d) IV fluid administration
39.  Which of the following is a symptom consistent with a diagnosis of Irritable bowel syndrome?
a) Painless diarrhea
b) Fever
c) Postprandial urgency
d) Steatorrhea

40. Which one of the following statement for a patient with Irritable bowel syndrome considered we
should refer the patient to specialist?
a) diarrhea alternating with constipation
b) rectal bleeding
c) who take ASPIRIN
d) none
41.According to ROME III criteria, for what length of time must symptom onset be for a diagnosis of
IBS?
A.9 months
B.4 months
C.12 months
D.6 months
E.3 months
42.Which of the following symptoms is not associated with Irritable bowel syndrome?
A. UrgencyMucousonstool
B. Meleana
C. Backache
D. Lowmood
43. Which of the following is clinupically diagnostic of Irritable bowel syndrome?
A. Improvementwithdefecation
B. Nonflushable/palestools
C. Bloodmixedwithstool
D. LIF pain
E. Reboundtenderness
44.Once other disease condition have been ruled out, a person can be considered for the diagnosis of
irritable bowel syndrome if the symptoms were present for the last
A. Oneweek
B. Onefortnight
C. Onemonth
D. Threemonths
45.Which psychological treatment option for irritable bowel syndrome (IBS) is supported by the best
evidence?
a) Cognitive–behavioralpsychotherapy
b) Supportgroupmeeting
c) A + B
d) none 
46. Which of the following is not recognized as a symptom that supports the diagnosis of Irritable
bowel syndrome according to the Rome criteria?
a) Altered stool frequency
b) Mucorrhea
c) Abdominal bloating or subjective distention
d) Frequent nausea

47.Which of the following symptoms indicates a need for laboratory testing or diagnostic imaging in
patients with IBS younger than 50 years?
a) Iron deficiency anemia
b) Abdominal pain
c) Amenorrhea
d) Hypokalemia

48.. Which of the following medications is indicated for the treatment of Irritable bowel syndrome?
a) Loperamide
b) Sertraline
c) Clozapine
d) Prednisone

49. A 24-year-old male was referred for a colonoscopy to investigate his symptoms of weight loss and
diarrhoea. The endoscopist thought that they could view skip lesions in the terminal ileum and took
several biopsies. What is the histology most likely to show?
a. Ulcerative colitis
b. Coeliac disease
c. Irritable bowel syndrome
d. Crohn’s disease
e. Bacterial enterocolitis

50. A patient gives chronic history of diarrhoea and blood in stool presents with multiple fistulae in the
perineum and multiple stricture in small intestine. The diagnosis is -
a) Crohn's disease
b) Radiation enteritis
c) Ulcerative colitis
d) Ischemic bowel disease
e) Peptic ulcer

51.An elderly male patient with prior history of hematemesis is having hx of long term use of aspirin
and other drugs, now presents with severe epigastric pain, dysphagia
and vomiting. He was connected to vital monitors which were not reassuring.
What is the management?
a. Oral antacids
b. IV PPI
c. Oral PPI
d. Endoscopy

52.Which drug is not effective against H.pylori-


a) ColloidalBismuth b) Metronidazole
c) Amoxicillin d) Erythromycin

53. A 74-year-old man complaining of bitter taste and substernal burning 30-40 minutes after meals.
The burning is relieved with antacids and worsened by lying supine.He has had an unintentional weight
loss of 4.5 kg (1 0 lb}over the past 3 months. Chest x-ray andelectrocardiogram show no abnormalities.
Which of the following is the most appropriate next step in management of this patient?
A.Upper gastrointestinal endoscopy
B. Esophageal pH monitoring
C. He/icobacterpyloristool antigen testing
D. Trial of famotidine
54 .A 56-year-old man presents to your office with complaints of “chronic”
diarrhea. He states that he has had loose stools for the past 2 days. He
denies blood in the stool, fever, and has no weight loss and no recent travel.
Appropriate management at this time includes
A) Observation
B) Check stool cultures
C) Colonoscopy
D) Stool fat studies

55. A 66-year-old woman comes to the physician with a 4-week history of abdominal symptoms.
Shecomplainsof upper abdominal discomfort and occasional dull epigastric pain accompanied by
nausea. Her symptoms are often worse after eating, especially with large meals. Which of the following
is the most appropriate next step in management of this patient?
A. Abdominal ultrasound
B. Antacids and metoclopramide
C. Helicobacter pylorystool antigen testing
D. Upper gastrointestinal endoscopy
56. A 69-year-old female presents to her GP with a two month history of indigestion. She is otherwise
well, has not had a similar episode before and takes no regular medication. Of note there is no recent
weight loss or vomiting and abdominal examination is unremarkable.
Whatisthemostappropriateinitialmanagement?
A. Long-term course of a H2 receptor antagonist
B. Urgent referral for endoscopy
C. One month course of a full-dose proton pump inhibitor
D. Urea breath testing and treat for H pylori if positive
57. A 69 year old smoker has had increasing dysphagia when eating solid food which has been on
going for the past 3 months. He has notice a drop of 8 kg in weight in the past few months. What
SINGLE investigations is most likely to lead to a diagnosis?
A. Barium swallow
B. Chest X-ray
C. Computed tomography chest
D. Endoscopy and biopsy
58. A 68 yo man has an intermittent epigastric pain for 3wks. It is worse by
food but helped by some tablets he obtained from the pharmacy. He had a
similar episode 3yrs ago and his doctor gave him a course of 3 types of tablets
at the time. What is the most appropriate next inv?
a. Abdomen US
b. Barium meal
c. Serum H.Pylori antibodies
d. Urea breath test
e. Upper GI endoscopy

59. A 38-year-old male visits his GP complaining of intermittent abdominal pain. This is epigastric in
nature and radiates through to the back. It is more severe before meal times and relieved by eating. A
C14 breath test is arranged which comes back positive. What is the most likely diagnosis?
a. Duodenal ulcer
b. GORD
c. Barrett’s oesophagus
d. Achalasia
e. Gastric ulcer
60. In the occurrence of peptic ulcer disease in elderly and senile patients, the most important are:
a) violation of microcirculation in the wall of the stomach;
b) genetic predisposition;
c) increase the acidity of gastric juice;
d) Helicobacter pylori infection.
61.For gastric ulcer and 12 duodenal ulcer in elderly and senile people it is characteristic:
a) lack of symptoms; predisposition to malignancy;
b) more severe course;
c) ulcers of large and gigantic sizes;
62.A characteristic feature of duodenal ulcer in old age is:
a) the occurrence of pain after 1-1.5 hours after a meal;
b) exacerbation of pain after pneumonia;
c) irradiation of pain in the left half of the chest;
d) heartburn.

63.A 45yo man, known to be chronically addicted to alcohol, presents in the ED and reports two
episodes of vomiting fresh bright red blood in the previous 6h. He estimated the volume blood vomited
at each bout to be more than 500mls. Clinical exam: the radial pulse=120/min, BP=90/60mmHg. There
is no mass or tenderness in the epigastrium. The liver is palpable for 3 cm below the costal margin and
not tender. The patient is not jaundiced. The physician resuscitates the patient with oxygen by face
mask, rapid infusion of intravenous normal saline while he requests for haemoglobin level and whole
blood for transfusion. Which is next appropriate step in management?
a. Barium Swallow
b. Exploratory laparotomy
c. CT scan of the abdomen
d. Upper gastrointestinal endoscopy
64. Which of the following is an acceptable treatment for Helicobacter pylori infection?
A) Bismuth, amoxicillin, metronidazole, and omeprazole
B) TMP-sulfamethoxazole, sucralfate, and metronidazole
C) Omeprazole, clindamycin, and sucralfate
D) Docusate, tetracycline, and metronidazole
E) Ranitidine, metronidazole, and ampicillin
65. A young woman complains of diarrhea, abdominal cramps and mouth ulcers. AXR shows
distended transverse colon with globet cell depletion on rectal biopsy. What is the most
probable dx?
a. Cohn’s diseases
b. Ulcerative colitus
c. IBS
d. Bowel obstruction
66.A 28 years old female presents with a 3m hx of diarrhea. She complains of abdominal discomfort
and
passing stool 20x/day. Exam=febrile. Barium enema shows cobblestone mucosa. What is the most
likely diagnosis?
a. Ameoba
b. Colon cancer
c. Ulcerative colitus
d. Cohn’s diseases

67.A 27-year-old man complains of episodic abdominal pain. The pain is concentrated in the
epigastrium and is gnawing in quality. It wakes him up during the night and is promptly relieved by a
glass of water and a piece of bread. He denies vomiting or diarrhea, but has experienced an occasional
"dark stool." He has no significant past medical history and does not take any medications.
His vital signs are with in normal limits. Physical examination shows mild
epigastric discomfort on deep palpation. Which of the following is the best option to provide long-term
symptom relief in this patient?
A. Four weeks of omeprazole
B. Antibiotics and pantoprazole
C. NSAIDs as needed
_ G. Cholecystectomy

68.A 28 yo female presents with a 3m hx of diarrhea. She complains of abdominal


discomfort and passing stool 20x/day. Exam=febrile. Barium enema shows
cobblestone mucosa. What is the
most likely dx?
a. Ameoba
B Colon Ca
d. Ulcerative colitis.
e.Crohn’s disease

69.A young woman complains of diarrhea, abdominal cramps and mouth ulcers.
AXR shows distended transverse colon with goblet cell depletion on rectal biopsy. What is
the most probable dx?
a. Crohn’s disease
b. Ulcerative colitis.
c. Bowel Ca
d. Bowel obstruction
e. IBS

70.A 28-year-old woman is diagnosed with constipation predominant irritable bowel syndrome. She
occasionally experiences spasms of pain in the left iliac fossa. Which one of the following is LEAST
likely to help her symptoms?
A. Mebeverine
B. Ispaghula
C. Methylcellulose
D. Lactulose

71.A 30-year-old woman is admitted to hospital with abdominal pain and diarrhoea. She has no past
medical history other than depression for which she takes citalopram. She smokes 20 cigarettes/day and
drinks 20 units of alcohol per week. Ileocolonoscopy shows features consistent with Crohn's disease
and she is treated successfully with glucocorticoid therapy. Which one of the following is the most
important intervention to reduce the chance of further episodes?
A. Infliximab
B. Stop drinking
C. Stop smoking
D. Mesalazine
E. Budesonide

72. A 35-year-old female presents with abdominal pain associated with bloating for the past 6 months,
Which one of the following symptoms is least associated with a diagnosis of irritable bowel syndrome?
A. Feeling of incomplete stool evacuation
B. Weight loss
C. Back pain
D. Lethargy

73.A 22-year-old man presents with a three week history of diarrhoea. He says his bowels have not
been right for the past few months and he frequently has to run to the toilet. These symptoms had
seemed to be improving up until three weeks ago. For the past week he has also been passing some
blood in the stool and reports the feeling of incomplete evacuation after going. Examination of his
abdomen demonstrates mild tenderness in the left lower quadrant but no guarding. What is the most
likely diagnosis?
A. Diverticulitis
B. Colorectal cancer
C. Crohn's disease
D. Ulcerative colitis
E. Infective diarrhea
74.A 31-year-old man with ulcerative colitis presents with a worsening of his symptoms. He is passing
around four loose stools a day which do not contain blood. He has also experienced some urgency and
tenesmus but is otherwise systemically well. What is the most appropriate management?
A. Rectal mesalazine
B. Oral metronidazole
C. Rectal corticosteroids
D. Observe with review in 7 days time
E. Oral loperamide

75. A 27-year-old woman with chronic left iliac fossa pain and alternating bowel habit is diagnosed
with irritable bowel syndrome. Initial treatment is tried with a combination of antispasmodics, laxatives
and anti-motility agents. Unfortunately after 6 months there has been no significant improvement in her
symptoms. According to recent NICE guidelines, what is the most appropriate next step?
A. Low-dose tricyclic antidepressant
B.Cognitive behavioural therapy
C. Refer for sigmoidoscopy
D. Trial of probiotics

76.A 29-year-old man is reviewed. Four weeks ago he presented with a one month history of bloody
diarrhoea. He was previously fit and well prior to this episode. When initially reviewed he was passing
on average four loose stools a day with some visible blood. Colonoscopy showed extensive
inflammatory changes consistent with ulcerative colitis. He was started on oral mesalazine and a review
appointment was made for today. He is still passing around four bloody stools a day although he
remains systemically well. What is the most appropriate course of action?
A. Add oral prednisolone
B. Stop oral mesalazine and start oral prednisolone
C. Rectal corticosteroids
D. Admit for intravenous corticosteroids
E. Add oral azathioprine

77.A 36-year-old man presents with dyspepsia. No alarm symptoms are present. This is his first
episode and he has no significant medical history of note. A test-and-treat strategy is agreed upon.
What is the most appropriate investigation to test for Helicobacter pylori?
A. Gastric aspiration + culture
B. CLO test (rapid urease test)
C. Stool culture
D. Hydrogen breath test
E. 13C-urea breath test

78.You review a 34-year-old man who has had ulcerative colitis for the past 20 years. He describes a
one week history of passing three bloody stools per day. Despite this he is eating well and denies
abdominal pain. Abdominal examination is unremarkable. What is this episode most likely to
represent?
A. Severe exacerbation of ulcerative colitis
B.Infective exacerbation of ulcerative colitis
C. Mild exacerbation of ulcerative colitis
D. Moderate exacerbation of ulcerative colitis
E. Colorectal cancer secondary to longstanding ulcerative colitis

79.A patient presents with gastrointestinal symptoms. Which one of the following features in the
history would be least consistent with making a diagnosis of irritable bowel syndrome?
A. Urgency to open bowels
B. Symptoms made worse by eating
C. 62-year-old female
D. Passage of mucous with stool
E. Bladder symptoms
80. A 32-year-old male patient with chronic diarrhea, constipation, and joint
pain presents with a recent hip fracture. The most likely diagnosis is:
A. Irritable bowel syndrome
B. Crohn's disease
C. Ulcerative colitis
D. Gastroenteritis
E. Celiac disease
81. Crohn’s disease is associated with which of the following?
A) Inflammation limited to the superficial layer of the bowel wall
B) The affinity to involve the rectosigmoid junction
C) Decreased risk of colon cancer
D) Continuous mucosal areas of ulceration that affect the anus
E) Fistula formation

82. The most appropriate next step in younger patients with new-onset
symptoms of heartburn, especially after meals or while recumbent with or
without regurgitation, is:
A. Perform EGD to rule out esophagitis.
B. Withhold treatment until evaluation for H. pylori is complete.
C. Empirically treat with H2 blockers or PPI.
D. Perform 24-hour gastric pH study.
E. Obtain a gastric biopsy.

83. Anemia that is seen in patients with chronic renal disease is usually
caused by insufficient
a. Iron stores
b. Vitamin B12
c. Renin levels
d. Erythropoietin levels
e. Folate stores

84.The most common cause of chronic renal failure is-


a. Diabetes mellitus
b. Hypertension
c. Glomerular diseases
d. Interstitial diseases

85.Which one of the following is not a feature of renal artery stenosis?

a. Hypertension responds well to drugs


b. Kidneys maybe asymmetrical
c. Atherosclerotic plaques are common
d. Serum creatinine may increase with ACE inhibitors

86. A 65yo woman with DM, HTN and normal kidney function underwent a total right hip
replacement. She had massive haemorrhage during the operation and was given 8 units of packed RBC.
The blood pressure dropped to 60/40 mm Hg for about two hours before it was corrected with blood
transfusion. Two days after the surgery the serum creatinine level rose to 4.2 mg/dl (normal <1.5
mg/dl), BUN was 50 mg/dl (normal 10-20 mg/dl) and potassium 5.0 mmol/L (normal 3.5-5.0 mmol/l).
There were brown granular casts in the urine sediment. Which is the most likely cause of this
complication?
a. Diabetic nephropathy
b. Malignant hypertension
c. Acute tubular necrosis
d. Interstitial nephritis
87.A 5-year-old boy presents to his GP with a 3-day history of puffy eyes. He
has been unwell with a coryzal illness for the last week. His mother states he
has had no new medications and no hayfever, allergies or asthma. On further
examination he has generalized oedema and scrotal oedema. He is tachycardiac
and has cool peripheries, no skin rashes or erythema. What is the most likely
diagnosis?
a. Periorbital cellulitis
b. Allergic reaction
c. Nephrotic syndrome
d. Nephrotic syndrome with hypovolaemia
e. C1 esterase deficiency
88. 28 year old male met with an accident and sustained severe crush injury. He is most likely develop
-
a. Acute renal failure
b. Hypophosphatemia
c. Hypercalcemia
d. Acute myocardial infarction

89. Which of the following conditions is often the first sign of amyloidosis?
a) Proteinuria
b) CHF
c) Cardiac arrhythmias
d) Rheumatoid arthritis
e) Night blindness
90. Hyperphosphatemia is most often caused by:
a) Hypoparathyroidism
b) Rhabdomyolysis
c) Renal failure
d) Tumor lysis syndrome
e) Hyperthyroidism

91. A 19-year-old man presents with malaise, nausea, and decreased urine output. He was previously
well, and his physical examination is normal except for an elevated jugular venous pressure (JVP) and
a pericardial rub. His electrolytes reveal acute renal failure (ARF). Which of the following findings on
the urinalysis is most likely in keeping with acute glomerulonephritis (GN)?
a) proteinuria
b) white blood cell casts
c) granular casts
d) erythrocyte casts
e) hyaline casts
92. A 64-year-old man presents with weight gain, shortness of breath, easy bruising, and legs welling.
On examination, his blood pressure is 140/80 mm Hg, pulse 100/min, JVP 4 cm, heart sounds normal,
and lungs are clear. There is a 3+ pedal and some periorbital edema. Investigations include a normal
chest x-ray (CXR), electrocardiogram (ECG) with low voltages, anemia, high urea and creatinine, and
4 g/day of protein in the urine. A renal biopsy, which shows nodular deposits that have an apple-green
birefringence under polarized light when stained with Congo red. Which of the following is the most
likely diagnosis?
a) amyloidosis
b) multiple myeloma
c) diabetic nephropathy
d) minimal change disease
e) immunoglobulin A (IgA) nephropathy

93. A 64-year-old man presents with symptoms of malaise, shortness of breath, edema, and no urine
output for 24 hours. His past medical history is not significant, and his only medication is daily aspirin.
On examination his JVP is 4 cm, heart sounds are normal, lungs are clear, and the abdomen is soft. A
Foley catheter is inserted into his bladder for 200 cc of urine, which is sent for urinalysis. His urine
output still remains low. Which of the following is the most appropriate initial diagnostic test?
a) renal ultrasound
b) blood cultures
c) urine cultures
d) inferior vena cavagram with selective renal
venogram
e) blood urea nitrogen (BUN)/creatinine Ratio

94. A42-year-old man notices leg and facial swelling but no other symptoms. His examination is
pertinent for 3+ pedal edema including periorbital edema. A 24-hour urine collection reveals 5 g of
proteinuria. Which of the following is the most likely diagnosis?
(A) sickle cell disease
(B) medullary sponge kidney
(C) radiation nephritis
(D) staphylococcal infection
(E) amyloid disease

95.The disease, which can lead to secondary renal amyloidosis:


a. Rheumatism
b. deformingosteoarthrosis
c. rheumatoid arthritis
d. gout
96. The most specific marker of renal function is-
a. Creatinine clearance
b. Insulin clearance
c.Blood urea
d. Serum creatinine
97.What group of antibiotics causes acute renal failure more often?
1) penicillins
2) cephalosporins
3) aminoglycosides
4) macrolids
98.The main pathogenetic mechanism of edema in nephrotic syndrome:
a. hydrostatic
b. decreased oncotic pressure
c. Metabolic
d. dishormonal
99.The normal rate of glomerular filtration by endogenous clearance is:
b) 1)80 – 120 ml/min
c) 2) 50 - 75 ml/min
d) 3) 125 – 145 ml/min
e) 4) 150 - 170 ml/min
100. Anaphylactic shock is most likely to cause which of the following types of AKI?
a) Prerenal
b) Instrinsic
c) Postrenal
d) Functional
101.Renal artery stenosis are most likely to cause which of the following types of AKI?
a) Prerenal
b) Instrinsic
c) Postrenal
d) Functional
102. Goodpasture syndrome is most likely to cause which of the following types of AKI?
a) Prerenal
b) Instrinsic
c) Postrenal
d) Functional
103. Glomerulonephritis is most likely to cause which of the following types of AKI?
a) Prerenal
b) Instrinsic
c) Postrenal
d) Functional

104. Hypercholesterolemia is commonly associated with:

a) diabetes mellitus;
b) hypothyroidism;
c) nephrotic syndrome;
d) nephritic syndrome;
105. What changes in proteinogram are characteristic of renal amyloidosis?
a)hypergammaglobulinemia
b)paraproteinemia
c)hyperalbuminemia
d)hyperproteinemia
106.In case of nephrotic syndrome in diabetic nephropathy with normal renal function the most
appropriate treatment is

a. Corticosteroids

b. Cytotoxics

c. Symptomatic treatment

d. Memranostabilisation therapy

e. Plasmapheresis

107.The most common type of AKI according to pathophysiology is


A diabetic.
)
B) prerenal.

C) postrenal.

D intrarenal.
)

108. In which one of the following conditions, is a renal biopsy contraindicated-


a) Acute renal failure
b) Uncontrolled hypertension
c) Nephrotic syndrome
d) Isolated hematuria
109.CRF with anemia best treatment -
Iron
a) Erythropoietin
С) Bloodtransfusion
В) Cyanocobalamine
110.All of the following are components of nephrotic syndrome except -
a) Edema b) Hypercoagulability
c) Hypocholesterolemiad) Infection

111. Which of the following statements regarding aminoglycosides is true?


A) Liver function should be followed closely during administration.
B) Volume of distribution is increased in obese patients.
C) Nephrotoxic effects can occur with administration.
D) Lupus-like syndrome can occur with prolonged use.
E) Respiratory depression is not associated with aminoglycosides.

112.In CRF there is-


a) Oliguria
b) Polyuria
c) Olijuria during day and polyuria during night
d) Anuria
113.Features of glomerular haematuria-
a) Dysmorphic R.B.C., R.B.C. cast

b) Fragmeneted R.BG.
c) Full of R.B.C. in high power field
d) Grosshaematuria

114. Metabolic complication in chronic renal failure include all of the following except -
a) Hyperkalemia b) Hypophosphatemia
c) Hypocalcemia d) Hypokalemia

115.Most common cause of nephrotic range proteinuria in an adult is -


a) Diabetes Mellitus
b) Amyloidosis
c) Hypertensive nephropathy
d) Wegner's Granulomatosis

116.The most common cause of chronic renal failure is-


a) Diabetes mellitus b) Hypertension
c) Glomerular diseases d) Interstitial diseases

117. A young boy presented with bilateral periorbital edema, ankle swelling and increase in body
weight. What is the most likely dx?
a. Chronic heart failure
b. Nephrotic syndrome
c. Renal failure
d. Acute heart failure
e. Glomerulonephritis
118.A 60yo man complains of tiredness, lethargy and itching that is severe after a hot bath. He also has
nocturia, polyuria and nausea and vomiting. Exam: pallor, pigmentation and generalized
edema. What is the single most likely dx?
a. Hyperthyroidism
b. Chronic renal failure.
c. Liver failure
d. Eczema
119.Which one of the following is not a feature of renal artery stenosis?
a) Hypertension responds well to drugs
b) Kidneys maybe asymmetrical
c) Atherosclerotic plaques are common
d) Serum creatinine may increase with ACE inhibitors

120.Renal vein thrombosis is/are caused by -


a) Nephrotic syndrome
b) Invasive renal cell carcinoma
c) Pregnancy
d) Dehydration
e) Trauma

121.All of the following are uremic manifestation improve with dialysis except -
a) Metabolic acidosis
b) Osteodystrophy
c) Asterixis
d) Nausea,vommiting and anorexia

122.Microalbuminuria refers to urinary albumin excretion rate of -


a) 30-300mg/24 hour b) 400-600 mg/24 hour
c) 700-900 mg/24 hour d) >1000 mg/24 hour
123.The absolute indications for dialysis include the following except-
a) Persistent or severe hyperkalaemia
b) Congestive cardiac failure
c) Pulmonary odema
d) Hyperphosphataemia
e) Severe acidosis
124.The accurate diagnostic aid in renal artery stenosis is -
a) Selective renal angiography
b) Ultrasound
c) CT scan
d) IVU

125.The term end stage renal disease (ESRD)' is considered appropriate when GFR falls to-
a) 50% of normal b) 25% of normal
с) 10 - 25% of normal d) 5 -10% of normal
126.All are features of Acute Renal Failure (ARF) except- a) Hypotension b) Metabolic acidosis
c) Hyperkalemia d) Hypertension

127.CRF with anemia best treatment -


a) Iron
b) Erythropoietin
c) Blood transfusion ;
d) Folic acid
128.A 28 year old boy met with on accident and sustained severe crush injury. He is most likely to
develop:
a) Acute Renal Failure
b) Hypophosphatemia
c) Hypercalcemia
d) Acute Myocardial Infarction

129.TSH cannot be used for monitoring response to


treatment in:
a. Primary hypothyroidism
b. Secondary hypothyrodism
c. Thyroprivic hypothyroidism
d. Iodine deficiency

130. A 46-year-old woman complains of increasing fatigue and muscle weakness over the past 6
months. She reports an inability to
concentrate at work and speaks with a husky voice. The patient
denies drug or alcohol abuse. Physical examination reveals cold
and clammy skin, coarse and brittle hair, boggy face with puffy
eyelids, and peripheral edema. There is no evidence of goiter or
exophthalmos. Laboratory studies show reduced serum levels
of T3 and T4. Which of the following is the most likely underlying cause of these signs and symptoms?
(A) Amyloidosis of the thyroid
(B) Autoimmune thyroiditis
(C) Thyroid follicular adenoma
(D) Multinodular goiter
(E) Papillary carcinoma of the thyroid

131. A 42-year-old woman presents with amenorrhea and emotional disturbances. You note upper
truncal obesity and suspect Cushing syndrome. Laboratory studies reveal elevated serum levels of
corticosteroids that can be lowered by administration of dexamethasone. Which of the following is the
most likely cause of hypercortisolism in this patient?
(A) Adrenal cortical adenoma
(B) Adrenal cortical carcinoma
(C) Adrenal cortical hyperplasia
(D) Pheochromocytoma
(E) Pituitary adenoma
132. Treatment of choice in Thyrotoxicosis:
a . Propranolol
b. Anti thyroid drugs
c. Surgery
d. Propranolol followed by surgery

133. Dancing carotid sign is seen in:


a. Thyrotoxicosis
b. Papillary Ca
c. Follicular Ca
d. Hashimoto’s disease
134. A 62-year-old woman presents to your clinic complaining of fatigue and lethargy over a period of
6 months. She cannot recall exactly when these symptoms started, but feels that they are worsening
with time. She describes dry skin and has noted that she is losing hair. On examination she is mildly
bradycardic at 52 beats/min with normal blood pressure and has dry, coarse skin. There are areas of
alopecia and mild lower extremity edema is noted. Which of the following is the most likely clinical
diagnosis and which test would be indicated for screening for the diagnosis?
A. Hyperthyroidism: thyroid-stimulating hormone (TSH)
B. Hyperthyroidism: unbound T4
C. Hypothyroidism: TSH
D. Hypothyroidism: unbound T4

135.A patient of chronic Left lung abscess with generalized edema, hypoproteinemia, hepato-
splenomegaly without renal failure but reduced urine output. Diagnosis is?
a. Amyloidosis
b. Chronic cor-pulmonale with Rt. Heart failure
c. Bronchiectasis
d. Bronchogenic carcinoma

136.The most reliable investigation in amyloid disease is:


a. Rectal biopsy
b. Immunoglobulin assay
c. Ultrasound
d. Abdominal fat pad biopsy

 
137. Acromegaly is associated with which of the following factors?
a. Excessive cortisol secretion
b. Lack of adequate parathyroid hormone
c. Excessive growth hormone
d. Thyroid dysfunction
e. Excessive gastrin secretion
138. In a 15-year-old with diabetes mellitus type 1, which of the following
findings is inconsistent with diabetic ketoacidosis?
a. Hypokalemia
b. Hyperglycemia
c. Hypophosphatemia
d. pH of 7.2
e. Hypermagnesemia

139. A 60-year-old woman is referred to your office for evaluation of hypercalcemia of 12.9 mg/dL.
This was found incidentally on a chemistry panel that was drawn during a hospitalization for cervical
spondylosis. Despite fluid administration in the hospital, her serum calcium at discharge was 11.8
mg/dL. The patient is asymptomatic. She is otherwise in good health and has had her recommended
age-appropriate cancer screening. She denies constipation or bone pain and is now 8 weeks out from
her spinal surgery. Today, her serum calcium level is 12.4 mg/dL, and phosphate is 2.3 mg/dL. Her
hematocrit and all other chemistries including creatinine were normal. What is the most likely
diagnosis?
a. Breast cancer
b. Hyperparathyroidism
c. Hyperthyroidism
d. Multiple myeloma
e. Vitamin D intoxication

140. A 62-year-old man presents to a local emergency room complaining of chest pressure and feeling
“like my heart is fluttering inside my chest.” He experienced similar symptoms 1 month ago that
resolved spontaneously. He did not seek medical attention at that time. He has no significant past
medical history. On review of systems he notes some recent weight loss and excessive sweating. He
feels as though his appetite has increased lately. His wife adds that he has recently taken some time off
work due to fatigue; despite his time off he has not been able to relax and has not been sleeping well.
On physical examination his heart rate is irregular at 140–150 beats/minute. Blood pressure is 134/55
mmHg. He is admitted to the hospital and screening tests reveal an undetectable thyroid-stimulating
hormone level. Which of the following statements is true?
a. 50% of hyperthyroid patients will convert from atrial fibrillation to
normal sinus rhythm with thyroid management alone.
b. A firm, small thyroid on physical examination would be compatible with
a diagnosis of Graves’ disease.
c. Atrial fibrillation is the most common cardiac manifestation of
hyperthyroidism.
d. His excessive sweating is likely not related to hyperthyroidism.
e. Hyperthyroidism leads to a high-output state for the heart, and narrowing
pulse pressure.

141. The patient described above is started on atenolol and his heart rate slows to 80 beats/min. Which
of the following additional therapies is indicated?
a. Diltiazem
b. Itraconazole
c. Liothyronine
d. Methimazole
e. Phenoxybenzamine
142. A 35yo woman presents with a swelling in the neck. The swelling has increased in size gradually
over the last two years and the patient feels she has difficulty with breathing. Exam: mass measures
8cm by 10 cm, soft and not warm to touch. It moves with deglutition. Which is the most appropriate
management of this mass?
a. Partial thyroidectomy
b. Oral thyroxine
c. Oral propylthiouracil
d. Excision biopsy
143.You are reviewing the blood results of a 45-year-old obese man who had been complaining of
tiredness. His full blood count, urea and electrolytes and thyroid function tests were within normal
limits. The fasting plasma glucose result is shown below: Fasting plasma glucose 6.2 mmol/l What is
the most appropriate interpretation of this result?
a. Consistent with diabetes mellitus - need to confirm with a repeat sample
b. Normal - no further action needed
c. Borderline - repeat sample in 12 months
d. Impaired glucose tolerance - moderate risk of developing type 2 diabetes
mellitus
e. Prediabetes - high risk of developing type 2 diabetes mellitus
144. You review a 67-year-old man with type 2 diabetes mellitus in the diabetes clinic. His blood
pressure is currently 150/86 mmHg. His diabetes is well controlled and there is no evidence of end-
organ damage. What should his target blood pressure be?
a. < 140/80 mmHg
b. < 125/75 mmHg
c. < 140/85 mmHg
d. < 130/80 mmHg

145. An obese 48-year-old man presents with lethargy and polydipsia. What is the minimum HbA1c
that would be diagnostic of type 2 diabetes mellitus?
a. Cannot use HbA1c for diagnosis
b. 6.0% (42 mmol/mol)
c. 6.3% (45 mmol/mol)
d. 6.5% (48 mmol/mol)
e. 7.0% (53 mmol/mol)
146. A 43-year-old woman presents for follow-up in clinic. She was diagnosed with Hashimoto's
thyroiditis four months ago and is currently being treated with levothyroxine 75 mcg od. What is the
single most important blood test to assess her response to treatment?
a. ESR
b. TSH
c. Free T4
d. Total T4
e. Free T3
147.A 33-year-old female is referred by her GP with thyrotoxicosis. Following a discussion of
management options she elects to have radioiodine therapy. Which one of the following is the most
likely adverse effect?
a. Hypothyroidism
b. Thyroid malignancy
c. Agranulocytosis
d. Oesophagitis
e. Precipitation of thyroid eye disease
148.A 40-year-old woman complains of feeling tired all the time and putting on weight. On
examination a diffuse, non-tender goitre is noted. Blood tests are ordered: TSH 15.1 mU/l Free T4 7.1
pmol/l ESR 14 mm/hr Anti-TSH receptor stimulating antibodies Negative Anti-thyroid peroxidase
antibodies Positive
What is the most likely diagnosis?
a. Pituitary failure
b. Primary atrophic hypothyroidism
c. De Quervain's thyroiditis
d. Hashimoto's thyroiditis
e. Grave's disease

149.A 52-year-old man has a set of fasting bloods as part of a work-up for hypertension. The fasting
glucose comes back as 6.5 mmol/l. The test is repeated and reported as 6.7 mmol/l. He says he feels
constantly tired but denies any polyuria or polydipsia. How should these results be interpreted?
a. Impaired fasting glycaemia
b. Suggestive of diabetes mellitus but not diagnostic
c. Diabetes mellitus
d. Normal
e. Impaired glucose tolerance

150.A patient is diagnosed with type 2 diabetes mellitus. Following NICE guidelines, what target
should be set for the HbA1c?
a. Agree target with patient but generally aim for 7.0%
b. Agree target with patient but generally aim for 6.0%
c. As low as possible
d. Agree target with patient but generally aim for 6.5%

151.A 65-year-old man with a history of ischaemic heart disease is admitted with chest pain. The 12-
hour troponin T is negative. During admission his medications were altered to reduce the risk of
cardiovascular disease and to treat previously undiagnosed type 2 diabetes mellitus. Shortly after
discharge he presents to his GP complaining of diarrhoea. Which one of the following medications is
most likely to be responsible?
a. Gliclazide
b. Clopidogrel
c. Rosiglitazone
d. Metformin
e. Atorvastatin

152. A 54-year-old woman presents to your office with complaints of


frequent sweating episodes, palpitations, nervousness, and sensitivity to
heat with increased appetite and weight loss. The most likely diagnosis is
a. Hypothyroidism
b. Menopause
c. Addison’s disease
d. Hyperthyroidism
e. Cushing’s disease

153. Which of the following statements about hyperglycemic hyperosmolar


nonketotic coma is true?
a. It is usually associated with type I adult-onset diabetes mellitus.
b. It is associated with fluid overload.
c. Associated laboratory findings include elevated serum lactate.
d. Treatment involves intravenous administration of glucose.
e. Treatment involves fluid administration.
154.A 29 yo male brought to ED in unconscious state. There is no significant past hx. Which of the
following should be done as the initial investigation?
a. CT
b. Blood glucose
c. CBC
d. MRI

155. Clinical signs of acute adrenal failure are:


a) sharp fall of arterial pressure, dyspepsia
b) sharp elevation of arterial pressure
c) hyperpigmentation of skin
d) elevation of body temperature, fever

156.Emergency measures in acute adrenal failure include:


a) glucocorticoids, mineralocorticoids
b) sulfonamides
c) antibiotics, sulfonamides
d) beta-adrenergic blocking agents

157. Corticosteroma (Itsenko-Cushing syndrome) is characterized by two signs:


a) low level of blood cortisol, high level of ACTH in blood
b) high level of blood cortisol, low level of ACTH in blood
c) normal blood cortisol, normal CBC
d) high level of ACTH (adrenocorticotropic hormone) in blood

158. Which of the following statements regarding autoimmune hypothyroidism is true?


A. 10% of 40- to 60-year-old adults have subclinical hypothyroidism.
B. Absence of a goiter makes autoimmune hypothyroidism unlikely.
C. Family history of autoimmune disorders does not significantly increase risk.
D. It is more common in the Pacific Rim where diets are lower in iodine.
E. Viral thyroiditis does not induce subsequent autoimmune thyroiditis.

159. A 38-year-old woman presents to her primary care doctor complaining of fatigue and irritability.
She thinks these symptoms have been worsening over a period of several months. She has a history of
mild intermittent asthma and hypertriglyceridemia. Physical examination reveals a resting heart rate of
105 beats/min, blood pressure of 136/72 mmHg, bilateral proptosis and warm, moist skin. Screening
tests are sent and reveal a thyroid-stimulating hormone (TSH) level that is undetectable and a normal
unbound T4. What should be the next step in diagnosis?
A. Radionuclide scan of the thyroid
B. Thyroid-stimulating antibody screen
C. Thyroid peroxidase (TPO) antibody screen
D. Total T4
E. Unbound T3
160. Which of the following statements regarding hormone release from the anterior pituitary is true?
A. All hormones are released in a pulsatile manner.
B. Follicle-stimulating hormone (FSH) and luteinizing hormone (LH) release are suppressed prior to
puberty and after menopause.
C. Somatostatin acts in a feedback loop to inhibit adrenocorticotropin hormone (ACTH) release.
D. Thyroid-stimulating hormone (TSH) is released primarily at night.
E. With the exception of prolactin, none of the anterior pituitary hormones are present in a fetus until
week 28 of gestation.

161. In the management of diabetic ketoacidosis:


a. Intracellular water deficit is best restored using half
strength saline [0.45%saline]
b. Potassium should be given even before checking the
serum potassium concentration
c. Bicarbonate infusion is often only necessary in
severe acidosis pH< 7.0
d. 5% dextrose solution should be avoided unless
hypoglycaemia supervenes
162. Cause of death in diabetic ketoacidosis in children?
a. Cerebral edema
b. Hypokalemia
c. Infection
d. Acidosis

163. Which is not seen in diabetic ketoacidosis:


a. Normal serum potassium
b. Plasma osmolality 380 mosm
c. Urine Rothera test positive
d. Urine Benedicts test positive

164. An obese patient presented in casualty with random


blood sugar 400 mg%, urine sugar +++ and ketones .Drug useful in management will be:
a. Glibenclamide
b. Troglitazsone
c. Insulin
d. Metformin
165. The most effective correction of acidosis in diabetic
ketoacidosis is by:
a. l.V. bicarbonate
b. I.V. saline
c. I.V. insulin
d. Oral bicarbonate

166.An elderly male present with polyuria, polydipsia and weight loss for 3 months. Which of the
following parameter is diagnostic of diabetes mellitus in this patient?
a. Fasting blood sugar 117 mg%
b. Fasting blood sugar 137 mg%
c. 2 hours blood sugar after OGT 180 mg%
d. Urine dipstic +++ [3+]

167. Most common oral infection in diabetes mellitus?


a. Candida
b. Aspergillus
c. Streptococcus
d. Staphylococcus

168.A 34-year-old man complains of sudden attacks of dizziness, nblurred vision, and excruciating
headaches of 4 months in duration. During one of these attacks, his blood pressure was 180/120 mm
Hg. The patient’s father had been treated for thyroid cancer about 15 years ago. Laboratory studies
show
normal serum levels of aldosterone, renin, and angiotensin.
A 24-hour urinalysis reveals increased metanephrines. Episodic hypertension in this patient is most
likely caused by a tumor in which of the following endocrine organs?
a. Adrenal
b. Kidney
c. Parathyroid
d. Pituitary
e. Thyroid
169.Patient N., 70 y/o, is complaining of stomach ache, nausea, vomiting and muscle ache.
Objectively: evident symptoms of dehydration, Kussmaul’s breathing, arterial pressure – 95/60
mm column of mercury, anuria, temperature – 35.9 ºC, glycemia – 11.6 mmol/l, acetonuria is not
present, blood PH – 6.7, content of lactic acid -1.9 mmol/l (norm - 0.62 -1.3 mmol/l). What is
your diagnosis?
A.Hyperlactacidemic coma
B. Uremic coma
C. Ketoacidotic coma
D. Brain coma
E. Hyperosmolar coma

170.Patient M, 72 y/o, is in the intensive care unit with the symptoms of dehydration, oliguria,
hypothermia, hypoxemia (hypoxia). In the anamnesis there is a record of type 2 diabetes mellitus
treated with biguanides. Her condition began to deteriorate after she had a myocardial infarction
one month ago. Objectively: the skin is dry; turgor is lowered, arterial pressure – 80/40 mm
column of mercury, pulse – 136beats/minute. The breathing is shallow, eye ball tone is lowered.
What is your diagnosis?
A. Hyperlactacidemic coma
B. Uremic coma
C. Ketoacidotic coma
D. Brain coma
E. Hyperosmolar coma

171. A major disadvantage of the use of sulfonylureas in patients with diabetes is:
A. Vitamin B12 malabsorption
B. Weight loss
C. Treatment cost
D. Risk of hypoglycaemia
E. Risk of urinary tract infections

172. John, a 60-year old male has type 2 diabetes mellitus for the past 20 years. He has moderate
retinopathy and albuminuria. He has peripheral neuropathy affecting his both lower limbs. His
glycosylated hemoglobin is 8.0% while on metformin 850 mg bd, glipizide 10 mg bd, sitagliptin
100 mg om and dapagliflozin 10 mg daily. What would your therapeutic consideration to
improve his diabetes outcomes?
A. Reduce HbA1c to 7% and below with insulin
B. Optimize hypertension and dyslipidemia control
C. Order a CT coronary artery
D. Start aspirin and high dose statin
E. Order blood pro-BNP to detect subclinical heart failure

173.Treatment of severely infected diabetic foot ulcers should involve


A) Topical antibiotics
B) Debridement only
C) Debridement with systemic antibiotics
D) Debridement with topical antibiotics
E) None of the above
174. Acromegaly is associated with which of the following factors?
A) Excessive cortisol secretion
B) Lack of adequate parathyroid hormone
C) Excessive growth hormone
D) Thyroid dysfunction
E) Excessive gastrin secretion
175. Which of the following statements about diabetic retinopathy is true?
A) Proliferative retinopathy is associated with a poorer prognosis than
nonproliferative retinopathy.
B) Nonproliferative retinopathy is associated with neovascularization.
C) Symptoms of retinopathy usually begin with eye pain.
D) Diabetics should have eye examinations every 3 years.
E) Unfortunately, there is no treatment for diabetic retinopathy.
176.Tufting of distal phalanx is characteristic of:
a. Psoriatic arthropathy
b. Gout
c. Hyperparathyroidism
d .Hypoparthyroidism

177.A 50-year-old male presents to the clinic for a routine health examination. A comprehensive
metabolic panel shows a serum calcium level of 11.2 mg/dL. Serum phosphate is 3.0 mg/dL.
Serum creatinine is normal. He denies bone pain, lethargy, weakness, or weight loss. What is the
most common cause of hypercalcemia in outpatients?
A. Malignancy
B. Medications
C. Milk-alkali syndrome
D. Primary hyperparathyroidism
E. Granulomatous disease

178. A 62-year-old woman presents complaining of joint pain, polyuria,


polydipsia, and generalized fatigue. The woman reports a history of
recurrent kidney stones and depression. Radiographs show osteopenia and
subperiosteal resorption on the phalanges. Which of the following blood
tests may best help determine the cause of her symptoms?
A) ACE level
B) Parathyroid hormone level
C) ANA test
D) Sedimentation rate (ESR)
E) Bone densitometry
179.Based on the USPSTF recommendation, screening DEXA in all woman
without fracture risk factors should start
A) At age 50
B) At age 55
C) At age 60
D) At age 65
E) At age 70

180. A 48-year-old woman presents to her primary care physician because of 2 weeks of neck
pain. The pain is constant and sharp (rated at 10 of 10) and is felt in the anterior portion of her
neck. She also notes several weeks of loose stools and fatigue. Past medical history is significant
for a viral upper respiratory infection about 1 month ago. She has a temperature of 37.9°C
(100.2°F), heart rate of 96/min, and blood pressure of 136/82 mm Hg. On neck examination
there is diffuse enlargement of the thyroid and it is exquisitely tender to even mild palpation.
Laboratory tests show a total triiodothyronine level of 280 ng/dL, total thyroxine of 25 μg/dL,
and thyroid-stimulating hormone of 2 μU/mL (normal: 0.4–4 μU/L).
Which of the following is the most likely diagnosis?
(A) Acute infectious thyroiditis
(B) Drug-induced thyroiditis
(C) Hashimoto’s thyroiditis
(D) Riedel’s thyroiditis
(E) Subacute granulomatous thyroiditis
181. An 18-year-old woman presents to the emergency department with acute mental status
changes, rapid and deep breathing, abdominal pain, and vomiting. On examination she is
tachypneic and tachycardic, her abdomen is soft and nontender, and her mucous membranes are
dry. Laboratory values are notable for a potassium level of 5.5 mEq/L, bicarbonate of 12 mEq/L,
and serum glucose of 400 mg/dL. Which of the following is the most appropriate strategy during
the fi rst 24 hours?
(A) Diuresis and ventilatory support
(B) Diuresis, strict potassium restriction, and insulin
(C) Intravenous fluids, insulin, and potassium
(D) Intravenous fluids, insulin, and strict potassium restriction

182. A 28-year-old patient with known Addison’s disease presents with abdominal pain and is
hypotensive to a systolic pressure of 88 mm Hg.
He has a 2-week history of progressively worse
nonproductive dry cough, sore throat, malaise,
and headache. He has not checked his temperature at home but complains of constant chills.
Which of the following is the best initial management?
(A) Azithromycin
(B) Check serum glucose
(C) Hydrocortisone
(D) Intravenous fluids
(E) X-ray of the chest

183. A 49-year-old woman presents to her physician’s office with a long-standing history of
polydipsia, polyuria, central obesity, and hyperlipidemia. She is currently taking metformin, a
sulfonylurea, and an angiotensin-converting
enzyme (ACE) inhibitor. ACE inhibitors are
mostbeneficial in preventing or slowing the
progression of which of the following diabetic
complications?
(A) Diabetic ketoacidosis
(B) Diabetic nephropathy
(C) Diabetic neuropathy
(D) Diabetic retinopathy
(E) Peripheral vascular disease
184.A 32-year-old G2P1 woman at 16 weeks’ gestation presents to her obstetrician complaining
of fatigue, anxiety, and palpitations. She
says she has been feeling warm, even in her
air-conditioned home, and has been having
three or four loose stools per day, as compared
to one or two prior to her pregnancy. She has
a temperature of 37.1°C (98.9°F), heart rate
of 105/min, and blood pressure of 128/76 mm
Hg. Neck examination reveals mild diffuse
enlargement of the thyroid gland with nolymphadenopathy. Relevant laboratory fi ndings include
a total triiodothyronine level of
400 ng/dL, free thyroxine of 6.8 ng/dL, and
thyroid-stimulating hormone of 0.01 μU/mL
(normal: 0.4–4 μU/L). Results of a thyroidstimulating hormone- receptor antibody test are
positive. Which of the following is the most appropriate therapy for this patient?
(A) High-dose iodine therapy
(B) Methimazole
(C) Propylthiouracil
(D) Radioiodine ablation
(E) Surgical resection

185. A 63-year-old woman presents for routine evaluation. She has had diabetes for the past 12
years
with complications of neuropathy and retinopathy. You decide to screen her for renal
complications of diabetes. Which of the following findings
is not compatible with diabetic nephropathy?
(A) nephrotic range proteinuria
(B) microalbuminuria
(C) hypertension
(D) red blood cell (RBC) casts in urine

186. A 63-year-old man becomes oliguric 2 days following an open cholecystectomy. Which of
the
following findings would suggest that prerenal
ARF is a major factor in the etiology?
(A) postural hypotension
(B) fractional excretion of sodium is 3%
(C) specific gravity is 1.012
(D) the urine sodium is 30 mEq/L
(E) heme-granular casts on urine microscopy
187. A 23-year-old woman presents to clinic complaining of months of weight gain, fatigue,
amenorrhea, and worsening acne. She cannot identify when her symptoms began precisely, but
she reports that without a change in her diet she has noted a 12.3-kg weight gain over the past 6
months. She has been amenorrheic for several months. On examination she is noted to have
truncal obesity with bilateral purplish striae across both flanks. Cushing’s syndrome is suspected.
Which of the following tests should be used to make the diagnosis?
A. 24-h urine free cortisol
B. Basal adrenocorticotropic hormone (ACTH)
C. Corticotropin-releasing hormone (CRH) level at 8 A.M.
D. Inferior petrosal venous sampling
E. Overnight 1 mg dexamethasone suppression test
188. A 34-year-old woman presents to your clinic with a variety of complaints that have been
worsening over the past year or so. She notes fatigue, amenorrhea, and weight gain. She states
that her primary physician diagnosed her with hypothyroidism several months ago, and she has
been faithfully taking thyroid hormone replacement. Her thyroid-stimulating hormone (TSH) has
been in the normal range over the last two laboratory checks. When her symptoms did not
improve on synthroid, she was sent to your clinic for further evaluation. A diagnosis of
panhypopituitarism is considered. All of the following are consistent with normal pituitary
function except
A. basal elevation of follicle-stimulating hormone (FSH) and luteinizing hormone (LH) in a
postmenopausal woman
B. elevation of aldosterone after infusion of cosyntropin
C. elevation of growth hormone after ingestion of a glucose load
D. elevation of cortisol after injection of regular insulin
E. elevation of TSH after infusion of thyrotropinreleasing hormone (TRH)

189. A 42-year-old woman is brought to the emergency room by ambulance for altered mental
status. The glucose level by fingerstick monitoring was below the measurement capabilities of
the monitor (<40 mg/dL). After 2 ampules of 50% dextrose, the patient’s fingerstick glucose
remains at 42 mg/dL. She remains unconscious and had a 1-min seizure while in transport. She
has no history of diabetes mellitus. Her family denies that she has been recently ill, but recently
she has been depressed. She works as a registered nurse on a medical floor of the hospital.
Which of the following tests would confirm an overdose of exogenous insulin?
A. Plasma glucose <55 mg/dL, plasma insulin >18 pmol/L, and plasma C-peptide levels
undetectable
B. Plasma glucose <55 mg/dL, plasma insulin >18 pmol/L, and plasma C-peptide levels >0.6
ng/mL
C. Plasma glucose <55 mg/dL, plasma insulin <18 pmol/L, and plasma glucagon <12 pmol/L
D. Plasma glucose <55 mg/dL, plasma insulin <18 pmol/L, and C-peptide levels undetectable

190.Biguanides (e.g., metformin) act to control hyperglycemia in type 2 diabetes


mostly by:
A. Decreasing insulin resistance in muscle, fat, and liver
B. Stimulating pancreatic beta cells to increase insulin output
C. Decreasing glucose production by the liver
D. Inhibiting intestinal enzymes that break down carbohydrates, delaying
carbohydrate absorption

191. Preoperative assessment of a 55-year-old male patient


going for coronary angiography shows an estimated
glomerular filtration rate of 33 mL/min per 1.73 m2 and
poorly controlled diabetes. He is currently on no nephrotoxic
medications, and the nephrologist assures you that
he does not currently have acute renal failure. The case is
due to begin in 4 h, and you would like to prevent contrast
nephropathy. Which agent will definitely reduce the
risk of contrast nephropathy?
A. Dopamine
B. Fenoldopam
C. Indomethacin
D. N-acetylcysteine
E. Sodium bicarbonate
192. A 65-year-old male with a diagnosis of ESRD secondary
to diabetes mellitus is being treated with peritoneal
dialysis. This is being carried out as a continuous
ambulatory peritoneal dialysis (CAPD). He undergoes
four 2-L exchanges per day and has been doing so for approximately
4 years. Complications of peritoneal dialysis
include which of the following?
A. Hypotension after drainage of dialysate
B. Hypoalbuminemia
C. Hypercholesterolemia
D. Hypoglycemia
E. Left pleural effusion
193. A 63-year-old male with a history of diabetes mellitus
is found to have a lung nodule on chest radiography.
To stage the disease further he undergoes a contrast enhanced
CT scan of the chest. One week before the CT
scan, his BUN is 26 mg/dL and his creatinine is 1.8 mg/
dL. Three days after the study he complains of dyspnea,
pedal edema, and decreased urinary output. Repeat BUN
is 86 mg/dL and creatinine is 4.4 mg/dL. The most likely
mechanism of the acute renal failure is
A. acute tubular necrosis
B. allergic hypersensitivity
C. cholesterol emboli
D. immune-complex glomerulonephritis
E. ureteral outflow obstruction
194. Which of the following is the most appropriate medication for the
treatment of hypertension in a diabetic patient?
A) β-Blocker
B) ACE inhibitor
C) Diuretic
D) CCB
E) α-Blocker

195. Impaired glucose tolerance is corresponds to the concentration of glucose in capillary blood
(mmol / l)
1) fasting<6.7 through 2 hours ≥ 7,8 and <11.1
2) fasting<6.0 through 2 hours ≥ 6,1 and <7.8
3) fasting<6.1 through 2 hours ≥ 7,2 and <11.1
4) fasting<6.1 through 2 hours≥ 7,8 and <11.1

196.An elderly male present with polyuria, polydipsia and weight loss for 3 months. Which of
the following parameter is diagnostic of diabetes mellitus in this patient?

a. Fasting blood sugar 117 mg%

b. Fasting blood sugar 137 mg%

c. 2 hours blood sugar after OGT 180 mg%

d. Urine dipstic +++ [3+]

е.Glycosylated hemoglobin (Hgb A1c)

197.A 37 years old, female patient presents with complaints of weight loss despite increased
appetite, heat intolerance, fatigue & weakness over the past few months. On examination, her
HR was 120/min. She is noted to have protrusion of eyeballs. What would be her probable
thyroid test report show?
A.T3 increased, T4 increased TSH normal
B. T3 deceased, T4 decreased, TSH increases
C. T3 increased, T4 increased, TSH decreased
D.. T3 decreased, T4 normal, TSH decreased

198. A 17-year old girl who was evaluated for short height was found to have an enlarged
pituitary gland. Her T4 was low and TSH was increased. Which of the following is the most
likely diagnosis?
A.Pituitary adenoma
B.TSH-secreting pituitary tumor
C. Thyroid target receptor insensitivity
D. Primary hypothyroidism

199. Symptoms such as very high body temperature, rapid heartbeat, and shortness of breath
describe:
A .Thyroid storm
B . Thyroid crisis
C . Thyroid failure
D . Thyroid death

200. Which of the following studies is most sensitive for detecting diabetic nephropathy?
A. Serum creatinine level
B. Creatinine clearance
C. Urine albumin
D. Glucose tolerance test
E. Ultrasonography

201. Dose of insulin in diabetic nephropathy:


a. Insulin dose should be increased in patient with
ESRD
b. Insulin dose should be decreased in patients with
ESRD
c. Insulin does not need change in ESRD
d. Add inhaled insulin to conventional administration

202.Cushing syndrome is commonly caused by:


a. Adrenal adenoma
b. Adrenal hyperplasia
c. Ectopic adrenal hormone production
d. Adrenal carcinoma

203. A 28-year- old lady has put on weight [10 kg over a


period of 3 years] and has oligomenorrhoea followed
byamenorrhoea for 8 months. The blood pressure is
160 / 100 mm of Hg . Which of the following is the most
appropriate investigation?

a . Serum electrolytes
b. Plasma cortisol
c. Plasma testosterone and ultrasound
d. T 3,T4 and TSH

204.A patient presents with hemoptysis and Cushingoid


features with a lack of dexamethasone suppression ,
the likely reason could be: [ NBE Pattern 2014 -15]
a. Adrenal hyperplasia
b. Adrenal adenoma
c. CA lung
d . Ectopic ACTH production

205. A common cause of Cushing's syndrome is:


a.Cancer producing ectopic ACTH
b. Pituitary adenoma
c. Adrenal tuberculosis
d . None of the above
206.The predominant symptom / sign of pheochromo -
cytoma is:
a. Sweating
b. Weight loss
c. Orthostatic hypotension
d . Episodic hypertension
207.A patient with pheochromocytoma would secrete
which of the following in a higher concentration?
a. Norepinephrine
b. Epinephrine
c. Dopamine
d . VMA
208.Which of the following antibodies is involved in
the tissue destructive process associated with
hypothyroidism in Hashimoto's and atrophic
thyroiditis?
a. Thyroperoxidase antibody
b. Thyroglobulin antibody
c. TSH receptor antibody
d. Thyroid stimulating antibody
209.A14 year old girl on exposure to cold has pallor of extermities followed by pain and
cyanosis. In later ages of life she is prone to develop
a) Systemic lupus erythematosis
b) Scleroderma
c) Rheumatoid arthritis
d) Histiocytosis

210.A 35 year old lady complains dysphagia, Raynaud’s phenomenon, sclerodactyly.


Investigations show antinuclear antibody. The likely diagnosis is -
a) Systemic lupus erythematosis
b) Systemic sclerosis
c) Mixed connective tissue disorder
d) Rheumatoid arthritis
211. c-ANCA is characteristic for -
a) PolyarteritisNodosa
b) R.P.GN
c) Henoch’s Schonlein Purpura
d) Wegeners granulomatosis

212. The presence of a “bamboo spine” on spine radiographs, elevated


ESR, and a positive test for HLA-B27 support the diagnosis of
A) Multiple myeloma
B) Reiter’s syndrome
C) Ankylosing spondylitis
D) Rheumatoid arthritis
E) Pott’s disease

213. Inflammation and necrosis of the muscular tissue supplied by small and
medium-size arteries is known as
A) Polyarteritisnodosa
B) Pyoderma gangrenosum
C) Polymyositis
D) Giant cell arteritis
E) Dermatomyositis

214.A 23yo female presents with paresthesias and loss of distal pulses in her arms. She is noted
to be hypertensive. She describes feeling unwell a month prior with fever and night sweats. What
is the most probable diagnosis?
a. Kawasaki disease
b. Takayasu arteritis
c. Raynaud’s phenomenon
d. Embolism

215. A 46-year-old woman with severe asthma presents with increasing weight and back pain for
9 months. The patient is taking corticosteroids for her asthma. An X-ray of the vertebrae will
likely reveal which of the following pathologic findings?
a.Bone infarct
b. Dislocation
c.Osteomalacia
d. Osteomyelitis
e.Osteoporosis
216. Of the following four modalities, which is the most sensitive for evaluation of osteoporosis?
A. Ultrasound
B. CT scan
C. Plain x-rays
D. DEXA scan
E. Serum calcium
217. Which of the following statements regarding osteoporosis is true?
A) Routine screening of women older than 65 years is not recommended.
B) Dual-energy X-ray absorptiometry (DEXA) scans result in more
radiation exposure than qualitative CT.
C) T scores are used to diagnose osteoporosis.
D) Plain X-rays are a good diagnostic test for assessment of osteoporosis.
E) Medicare will not pay for bone-density examination.

218.Which of the following statements regarding osteoporosis is true?


A) Routine screening of women older than 65 years is not recommended.
B) Dual-energy X-ray absorptiometry (DEXA) scans result in more
radiation exposure than qualitative CT.
C) T scores are used to diagnose osteoporosis.
D) Plain X-rays are a good diagnostic test for assessment of osteoporosis.
E) Medicare will not pay for bone-density examination.
219. Patient R, him test results, and her bone density is very low with her hip -score 3.2 standard
deviations below the mean (− 3.2). You diagnose her with osteoporosis.
Given that she is otherwise relatively healthy, what is the most appropriate initial therapy or her
osteoporosis?
A) Alendronate (Fosamax) 70 mg PO weekly
B) Estrogen (e.g., Premarin) 0.625 mg PO aily
C) eriparati e (Forteo) 20 g SC aily or 5 years
D) Zoleronicaci (Reclast) 5 mg IV every 3 months

220. A 60-year-old man who had been treated for lung cancer complains of a rash on his chest
and pain in his upper arms and
calves. He cannot raise his arms and climbs the stairs only
withdifficulty. A muscle biopsy shows perivascular infiltrates
of lymphocytes and plasma cells extending in between the
muscle fi bers. Immunofluorescence reveals immune complexes in the walls of intramuscular
blood vessels. Which of
the following is the most likely diagnosis?
(A) Becker muscular dystrophy
(B) Dermatomyositis
(C) Lambert-Eaton myasthenic syndrome
(D) Myasthenia gravis
(E) Toxic myopathy

221. A 29-year-old man complains of nasal obstruction, bloody


nose, cough, and bloody sputum. A chest X-ray displays cavitated lesions and multiple nodules
within both lung fi elds.
Urinalysis reveals 3+ hematuria and red blood cell casts.
Laboratory studies show anemia and elevated serum levels of
C-ANCA (antineutrophil cytoplasmic antibody). Peripheral
eosinophils are not increased. A renal biopsy exhibits focal
glomerular necrosis with crescents and vasculitis affecting
arterioles and venules. What is the appropriate diagnosis?
(A) Churg-Strauss syndrome
(B) Goodpasture syndrome
(C) Hypersensitivity vasculitis
(D) Polyarteritisnodosa
(E) Wegener granulomatosis

222. A 24-year-old man on chronic corticosteroid therapy for


severe asthma presents with a 6-month history of increasing
hip pain. This patient most likely exhibits symptoms of which
of the following metabolic bone diseases?
(A) Gaucher disease
(B) Osteomalacia
(C) Osteopetrosis
(D) Osteoporosis
(E) Paget disease

223. A 27-year-old woman presents with a recent


episode of coughing up some blood, frequent
nosebleeds, and now decreased urine output.
A nasal mucosa ulcer was seen on inspection.
Her urinalysis is positive for protein and red
cells consistent with a GN. The CXR shows
twocavitary lesions and her serology is positive for antineutrophil cytoplasmic antibodies
(ANCA). Which of the following is the most
likely diagnosis?
(A) Wegener’s granulomatosis
(B) bacterial endocarditis
(C) Goodpasture’s syndrome
(D) lupus erythematosus
(E) poststreptococcal disease

224. A 67-year-old woman presents to clinic after a fall on the ice a week ago. She visited the
local emergency room immediately after the fall, where hip radiographs were performed and
were negative for fracture or dislocation. They did reveal fusion of the sacroiliac joints and
coarse trabeculations in the ilium, consistent with Paget disease. A comprehensive metabolic
panel was also sent at that visit and is remarkable for an alkaline phosphatase of 157 U/L, with
normal serum calcium and phosphate levels. She was discharged with analgesics and told to
follow up with her primary care doctor for further management of her radiographic findings. She
is recovering from her fall and denies any long-standing pain or immobility of her hip joints. She
states that her father suffered from a bone disease that caused him headaches and hearing loss
near the end of his life. Which of the following is the best treatment strategy at this point?
A. Initiate physical therapy and non-weight bearing exercises to strengthen the hip.
B. No treatment; she is asymptomatic. Follow radiographs and laboratory findings every 6
months.
C. Prescribe vitamin D and calcium.
D. Start an oral bisphosphonate.
E. Start high-dose prednisone with rapid taper over 1 week.
225. Which of the following is the most common site for a fracture associated with osteoporosis?
A. Femur
B. Hip
C. Radius
D. Vertebra
E. Wrist

226. Which of the following statements is true concerning seronegative


spondyloarthropathies?
A. It is associated with the HLA-B17 gene.
B. Reiter's syndrome or reactive arthritis is almost never associated with extraarticular
symptoms.
C. Ankylosing spondylitis is a disease of older women associated with asymmetric
monoarthritis.
D. Arthritis may be the first manifestation of inflammatory bowel disease, especially
Crohn's disease.
E. Inflammation at bony sites of tendon, ligament, and fascial attachment
(enthesitis) is rarely seen with reactive arthritis.

227. The presence of a “bamboo spine” on spine radiographs, elevated


ESR, and a positive test for HLA-B27 support the diagnosis of
A) Multiple myeloma
B) Reiter’s syndrome
C) Ankylosing spondylitis
D) Rheumatoid arthritis
E) Pott’s disease

228. An 18-years-old boy presents with digital gangrene in third and fourth fingers for last 2
weeks. On examination the blood pressure is170/110 mm of Hg and all peripheral pulses were
palpable. Blood and urine examinations were unremarkable.
Antinuclear antibodies, antibody to double stranded I DNA and anti-neutrophil cytoplasmic
antibody were negative.The most likely diagnosis is:
a. Wegner's granulomatosis
b.Polyarteritisnodosa
c. Takayasu's arteritis
d. Systemic lupus erythematosus [SLE]
229. Bilateral upper limb pulse less disease is:
a. Giant Cell Arteritis
b. PolyarteritisNodosa
c. Aortoarteritis
d. HSP
230. Incorrect about takayasu arteritis:
a. Spares pulmonary artery
b. Renovascular hypertension
c. Blood pressure difference between left and right
limbs
d.Strongly positive mantoux
231. Takyasu arteritis mainly affects:
a. Pulmonary artery
b. Celiac artery
c. Subclavian artery
d. SMA

232. A 20-year-old woman presents with bilateral


maxillary sinusitis, palpable purpura on the legs and
hemoptysis. Radiograph of the chest shows a thinwalled
cavity in left lower zone.Investigations reveal
total leukocyte count 12000/mm3, red cells casts in
the urine and serum creatinine 3 mg/dl. What is the
most probable diagnosis?
a. Henoch.Schonlein purpura
b. Polyarteritisnodosa
c. Wegener's granulomatois
d. Disseminated tuberculosis
233.InTakayasu's arteritis there is:
a. Intimal fibrosis
b. Renal hypertension
c. Coronary aneurysm
d. All of the above

234.What is feature of temporal arteritis?


a. Giant cell arteritis
b. Granulomatous vasculitis
c. Necrotizing vasculitis
d. Leucocytoclastic Vasculitis
235.Which of the following are true about findings of Polyarteritisnodosa ?
a. There is tear in the lamina Dura
b. Micro Aneurysm formation in the large blood vessel
c. Nodules are formed in skin which are clinically
palpable
d. Chain of beads appearance
236.Giant cell arteritis causes which of the following in
the eye:
a .Episcleritis
b. Anterior ischemic optic neuropathy
c. Neuroparalytic keratitis
d . Band keratitis

237.A 30 - year - old male patient presents with complaints


of weakness in right upper and both lower limbs for
last 4 months. He developed digital infarcts involving
2nd and 3 rd fingers on right side and 5th finger on
leftside.On examination, BP was 160 / 140 mmHg,
all peripheral pulses were palpable and there was
asymmetrical examination showed proteinuria and
RBC- 10 - 15/ hpf with no casts. What is the most likely diagnosis?

a. Polyarteritisnodosa
b. Systemic lupus erythematosus
c. Wegner’s granulomatosi
d . Mixed cryoglobulemia

238.Treatment of choice in Wegner's granulomatosis is:


a. Cyclosporine
b. Cyclophosphamide
c. Steroids
d . Radiotherapy
239.Upper respiratory tract infections are common in :
a . Tetralogy of Fallot
b. Cystic fibrosis
c. Mitral Regurgitation
d . Wegener's granulomatosis

240.Cavitating lesion in lung is seen in:


a . Wegner's granulomatosis
b. PAN
c. SLE
d .Goodpasture's syndrome

241.Hepatitis B virus is associated with:


a . SLE
b. Polyarteritisnodosa
c. Sjoren's syndrome
d . Wegener's granulomatosis

242. Pulse absent in radial artery is seen in?


a. Coarctation of aorta
b. Aortic regurgitation
c. Takayasu 's arteritis
d. Dissection of Aorta

243. C-ANCA is associated with:


a. Wegener's Granulomatosis
b. Microscopic Polyangitis
c. Churg- Strauss Syndrome
d. PolyarteritisNodosa [PAN]
244.All of the following condition are associated with
granulomatous pathology, except:
a. Wegner's Granulomatosis [WG]
b. Takayasu Arteritis [TA]
c. PolyarteritisNodosa [Classic PAN]
d. Giant Cell Arteritis

245. A 25 years old female develops serous otitis media


of left ear with cough and occasional hemoptysis and
hematuria and epistaxis for one and half months. Her
Hemoglobin is 7 gm .B . P. > 170 / 100, ptoreinuria + + +,
RA positive [+ ve] and ANCA positive [ +ve], the likely
cause is-
a. Wegener’s granulomatosis
b. Rheumiatoid arthritis
c. Rapidly proliferative glomerulonephritis
d. Good pasteur's syndrome

246.An 18 year old boy presents with digital gangrene in


3rd& 4th fingers for last 2 weeks. On examination
the BP is 170/ 110 mm of Hg and all peripheral pulses
were palpable. Blood & Urine examination were unre ¬
markable. Antinuclear antibody, Antibody to ds DNA
& DNA & ANCA were negative. Most likely diagnosis is:
a. Henoch -Schonlein purpura
b. Polyarteritisnodosa
c. Wegener’s granulomatosis
d. Disseminated tuberculosis

247.A patient presents with melaena normal renal function, hypertension and mononeuritis
multiplex. The
most probable diagnosis is:
a. Classical polyarteritisnodosa
b. Microscopic polyangiitis
c. Henoch-Schonleinpurpura
d. Buerger's disease

248.Biopsy in PolyarteritisNodosa shows : \


a. Necrotizing arteritis
b. Atrophy
c. Granulomatous lesion
d. Ring lesion
249.An elderly female presents to the emergency
department with history of fever, headache and
double vision. Biopsy of temporal artery revealed
panarteritis . The most likely diagnosis is
a. Nonspecific Arteritis
b. PolyarteritisNodosa
c. Wegener's Granulomatosis
d. Temporal Arteritis

250. Goodpasture’s syndrome is associated with


A) Osteoporosis and renal lithiasis
B) Pathologic fractures and thyroiditis
C) Hepatitis and recurrent cystitis
D) Pulmonary hemorrhage and glomerulonephritis
E) Pica and angioedema

251.The investigation of choice for diagnosis of Giant cell


Arteritis is
a. Temporal Artery biopsy
b. Colour Doppler of Temporal Artery
c. CT Angiography
d . MRI
252.Bilateral upper limb pulse less disease is?
a. Giant cell Arteritis
b .PolyarteritisNodosa
c. Aortoarteritis
d. HSP

253. Reversed Coarctation is seen in :


a . Giant cell Arteritis
b. PolyarteritisNodosa
c. Takayasu Arteritis
d. Kawasaki Disease
254.Renal artery stenosis may occur in all of the following,
except :
a. Atherosclerosis
b. Fibromuscular dysplasia
c. Takayasu's arteritis
d .Polyarteritisnodosa

255. Screening test for sclerodema:


a. Anti - nuclear antibody
b. Ul - Ribonucleoprotein antibody
c. Anti - L.K.M antibody
d. Anti - topoisomerase antibody
256. A 20-year-old woman presents with bilateral conductive deafness, palpable purpura on the
legs and hemoptysis. Radiograph of the chest shows a thin-walled cavity in left lower zone.
Investigations reveal total leukocyte count 12,000/mm red cell casts in the urine and 12,000/mm
serum creatinine 3 ing/dL. What is the most probable diagnosis ?
a) Henoch-Schonlein purpura
b) Polyarteritisnodosa
c) Wegener’s granulomatosis
d) Disseminated tuberculosis

257. An 18-year-old boy presents with digital gangrene in third and fourth fingers for last 2
weeks. On examination the blood pressure is 170/110 mm of Hg and all peripheral pulses were
palpable. Blood and urine examinations were unremarkable. Antinuclear antibodies, antibody to
double stranded DNA and antineutrophil cytoplasmic antibody were negative. The most likely
diagnosis is :
a) Wegener’s granulomatosis
b) Polyarteritisnodosa
c) Takayasu’s arteritis
d) Systemic lupus erythematosus (SLE)

258.Woman presented with dysphagia and stiff fingers i


and leather like skin is diagnosed to have?
a. Buergers disease
b. Rheumatoid arthritis
c. Sceleroderma
d .Osteoarthrosis

259. Jaw tightness in proximal temporal arteries. Diagnosis -


a) Polyarteritisnodosa
b) Coarctation of aorta
c) Giant cell arteritis
d) Wegner’s granulomatosis

260. A 35 year old lady complains dysphagia , Raynaud's


phenomenon, sclerodactyly. Investigations show
antinuclear antibody. The likely diagnosis is :
a. Systemic lupus erythematosis
b. Systemic sclerosis
c. Mixed connective tissue disorder
d . Rheumatoid arthritis

261.A 14 year old girl on exposure to cold has pallor of


extremities followed by pain and cyanosis. In later
stages of life she is most prone to develop:
a . SLE
b. Scleroderma
c. Rheumatoid Arthritis
d .Dermatomyositis

262.Arthritis common with uveitis is:


a. RA
b. Ankylosing spondylitis
c. Still's disease
d. Reiter's disease
251. In anklyosing spondylitis joint involvement is least in?
a. Wrist and hand
b. Sacroiliac joint
c. Acromio-clavicular joint
d. Costochondral junction
263.Treatment of choice in seronegative spondylarthritis
is:
a. Phenylbutazone
b. Aspirin
c. Indomethacin
d. Corticosteroid

264. 35-year-old male presents with pain in the knee joints for the last two weeks. He also has
mild backache and stiffness. He had fever and diarrhoea four weeks previous to developing these
complaints. The patient is afebrile and has no diarrhoea. His laboratory investigations reveal
normal ESR and elevated CRP. What is the most probable diagnosis?
a) Ankylosing spondylitis
b) Rheumatoid arthritis
c) Enteropathic arthritis
d) Reactive arthritis

265.A20 - year - old woman presents with bilateral conductive deafness, palpable purpura on the
le and hemoptysis. Radiograph of the chest shows a thin - walled cavity in left lower zone.
Investi - gations reveal total leukocyte count 12000 / mm red cell casts in the urine and 12, 000 /
mm serum creatinine 3 ing / dL .
What is the most probable diagnosis?
a. Henoch -Schonlein purpura
b. Polyarteritisnodosa
c. Wegener's granulomatosis
d. Disseminated tuberculosis
266. Which of the following signs and symptoms are associated with
Sjögren’s syndrome?
A) Hepatomegaly, chronic rhinitis, and palmar erythema
B) Keratoconjunctivitis, parotid gland enlargement, and xerostomia
C) Confusion, tremors, and peripheral neuropathies
D) Polycythemia, leukocytosis, and negative rheumatoid factor
E) Hyperextensibility of joints, iriditis, and glossitis

267. HLA B27 is seen associated with -


a) Rheumatoid arthritis
b) Ankylosing spondylitis
c) Sjogren syndrome
d) Scleroderma

268.Most common organism associated with reactive arthritis is-


a) Staphylococcus
b) Shigella
c) Chlamydia
d) Yersinia

269. A patient complains of gingival bleedings,incraesed body temperature till 38`C,marked


general weakness.He feels himself sick for the recent 2weeks.Data of objective
examination,petectnia on the skin,by palpation enlarged submandibular axillary and femoral
lymph nodes.Data of CBC: erythrocythes-2,1.10 9/l,Hb-80g/l, thrombocytes-120000/l, Le-
18,6 .10`9/l, l%,stub-2%,segm-9%, lymph-5%, mon-3%, blast cels-80%, ESR-36mm/hr.Which
data of CBC directly proves diagnosis of acute leukaemia?
a. erythocytes 2,1.10`12/l

b. thrombocytes-120000/l

c. leukocytes 18,6.10`9/l

d. ESR-36mm/hr

e. Blastcells 80%

270.A patient complains of gingival bleedings,incraesed body temperature till 38`C,marked


general weakness.He feels himself sick for the recent 2weeks.Data of objective
examination,petectnia on the skin,by palpation enlarged submandibular axillary and femoral
lymph nodes.Data of CBC: erythrocythes-2,1.10 9/l,Hb-80g/l, thrombocytes-120000/l, Le-
18,6 .10`9/l, l%,stub-2%,segm-9%, lymph-5%, mon-3%, blast cels-80%, ESR-36mm/hr.Which
disease has developed in the patient:
a. acuteleukaemia

b. chronicleukaemia

c. hemolyticanaemia

d. vitamin B12-deficiency anaemia

e. symptomofdysseminatedhypercoagulation

271. The first-line drug therapy in chronic myeloid leukemia is -


a) Hydroxycarbamide b) Alpha-interferon
c) Busulphan d)Imatinib

272.Patient, 62yrs old, is hospitalized with complaints of enlargement of neck, subclavian and
axillary lymph nodes, general weakness, increased sweating, subfebrile body temperature within
the last 3months. Data of CBC: WBC-64.109/l, lymphocytes-72%. Blast cells 1-2%. Which
disease is possible to suspect?
f. vitamin B12-deficiency anemia

g. plasmacellmyeloma

h. acutreleukaemia

i. chronicleukaemia

j. mononucleosis

273. The minimum number of blasts present in Bone marrow, for “acute leukemia” diagnosis is

A. 10%

B. 20%

C. 30%

D. 35%

274. A 55 year old male presents with enlarged glands over the left side of neck. On
examination, Spleen is enlarged 4 cm below the costal margin and liver is enlarged 2 cm below
costal margin. Blood examination show a total leucocyte count of 80,000/ cumm. Mostly
lymphocytes and a few prematurecells the mostly likely diagnosis is -
a) Infectiveadenopathy

b) Acuteleukaemoidreaction

c) Lymphaticleukaemia

d) Hodgkin’sdisease

275. A four year old boy was admitted with a history of abdominal pain and fever for two
months maculopapular rash for ten days, and dry cough, dyspnea and wheezing for three days.
On examination liver and spleen were enlarged 4 cm and 3 cm respectively below the costal
margins. Jjps hemoglobin was 10.0 g/dl, platelet count 37 x 10 FL and total leukocyte count 70 x
109/l, Bone marrow examination revealed a cellular marrow comprising of 45% blasts and. The
blasts stained negative for myeloperoxidase and non-specific esterase and were positive for
CD19, CD10, CD22 and CD20. Which one of the following is the most likely diagnosis?
a) Biphenotypic acute leukemia (lymphoid and eosinophil lineage)
b) Acuteeosinophilicleukemia

c) Acute lymphoblastic leukemia

d) Acute myeloid leukemia with eosinophilia

276.Which of the following findings is associated with chronic myelocytic leukemia (CML)?
A) Leukopenia
B) Philadelphia chromosome
C) Elevated leukocyte alkaline phosphatase level
D) Thrombocytopenia
E) Decreased vitamin B12 levels

277. A 65-year-old man presents complaining of back pain and generalized fatigue. Laboratory
findings include anemia with Rouleau formation, a monoclonal spike seen with serum protein
electrophoresis, and hypercalcemia. Radiographs of the lumbar spine show lytic lesions. The
most likely diagnosis is
A) Metastatic prostate cancer
B) Paget’s disease
C) Osteitisfibrosacystica
D) Multiple myeloma
E) Colon cancer

278.A 80-year-old male presents with tiredness, weakness and deteriorating confusion. On
examination he is pale and confused, has ataxia on standing with loss of vibration sensation and
exaggerated leg reflexes. A full blood count reveals a pancytopaenia. Select the most appropriate
treatment for the following cases with anaemia:

A. Parenteral Iron
B. Vitamin B12 injection
C. Blood Transfusion
D. Folate tablets
E. Oral Iron

279. Iron supplementations are prescribed:

1) for 1-2 weeks; 2) for 3-6 month, 3) for 2-3 weeks; 4) for 1 month
280.A 60-year-old male has moderate anemia, with a suggestion of hemolysis on a peripheral
blood smear. Which one of the following patterns would be consistent with the presence of
hemolysis?
a) Elevated LDH, decreased haptoglobin, elevated indirect bilirubin
b) Elevated LDH, elevated haptoglobin, decreased indirect bilirubin
c) Decreased LDH, elevated haptoglobin, elevated indirect bilirubin
d) Decreased LDH, decreased haptoglobin, elevated indirect bilirubin

281. Diagnosis of beta thalassemia is established


by:
A. NESTROFT test.
B. HbA1c estimation.
C. Hb electrophoresis.
D. Target cells in peripheral smear.

282.A 19-year-old Nigerian presents with painful haemolytic crises following a chest infection.
She is noted to have fingers of varying lengths.
A. hereditary spherocytosis
B. sickle cell anaemia
C. b-thalassaemia major
D. malaria

283. A 6-year-old boy has had severe anaemia since he was 6 months old. He has bossing of his
skull with enlarged maxilla, and prominent frontal and parietal bones. Choose the SINGLE most
likely cause

A. hereditary spherocytosis
B. sickle cell anaemia
C. b-thalassaemia major
D. malaria
284. A 60-year-old lady complains of tiredness, sore tongue, and numb/tingling feet. Full blood
count reveals a haemoglobin concentration of 7g/dL (11.5-16.5) and an MCV of 115 fL (80-96).
Choose the SINGLE most likely cause
A. hereditary spherocytosis
B. pernicious anaemia
C. disseminated intravascular coagulation
D. haemolyticanaemia

285. A 55-year-old male presents with a month history of tiredness and lethargy. He has been
hypertensive for 5 years and recently started alpha-methyldopa. Full blood count reveals a
haemoglobin of 8.5 g/dL (13-18) with an MCV of 92 fL (80-96) and an increased reticulocyte
count. Choose the SINGLE most likely cause

A. hereditary spherocytosis
B. pernicious anaemia
C. disseminated intravascular coagulation
D. haemolyticanaemia
286. A 24 year old woman with a seizure disorder treated with phenytoin is noted to have the
following findings at 15 weeks of pregnancy:

Hemoglobin (Hgb) 93 g/L (Normal 123-157 g/L)


Hematocrit (HCT) 29% (Normal 37-46%)
Mean corpuscular volume (MCV) 105 fL (Normal 80-100)

Which one of the following is the most likely cause for these findings?
a) Sickle cell trait
b) Iron deficiency
c) Physiologic anemia
d) Folate deficiency
e) Thalassemia

287.A 30 year old female presents to her physician's office for an increase in fatigue over the last
2 months. On physical examination, the physician notes delayed capillary refill in the nailbeds
and a pale appearance of the inner lining of the eyelids. A peripheral smear reveals hypochromic
erythrocytes. Which one of the following tests would be most useful in confirming the probable
diagnosis?

a) Serum ferritin and total iron-binding capacity (TIBC)


b) Schilling's test
d) Hemoglobin electrophoresis
e) Serum folate and cobalamin (vitamin B12)

289. Pathological bone fractures are most common with the following disease:
1. chronicmyelogenous leukemia
2. hemophilia
3.multiple myeloma
4. myelodysplastic syndrome

290. A 30-year-old woman presents to the gynaecology clinic with menorrhagia of two years
duration and also complains of tiredness and lethargy.Hemogramm:
Hemoglobin 6.8 g/l , Erythrocytes 3,2х1012/l , CI-0.7
Choose the SINGLE most likely cause
A. hereditary spherocytosis
B. sickle cell anaemia
C. iron deficiency anaemia
D. malaria
291.A 56-year-old woman had terminal ileum resection due to Crohn’s disease and was lost to
follow up. She presents on this occasion with difficulty walking. She is anaemic and has lost
position sense bilaterally.
A. hereditary spherocytosis
B. sickle cell anaemia
C. iron deficiency anaemia
D. vitamin B12 deficiency anaemia

292.A 50-year-old woman with a long history of alcohol abuse is prescribed Phenytoin for
epilepsy. Her full blood count reveals:
Haemoglobin10.0 g/dL, MCV-122 fL, White cell count-2.2 x109/L, Platelet count-85 x109/L
What is the most likely explanation for these results?
A. Alcoholic liver disease
B. Aplastic anaemia
C. Folic acid deficiency
D. Hypothyroidism
293. A 6-year-old girl presents with pallor and bruising. Haemoglobin 7.3 g/dl, White cell count
2.3x109/l, Platelets 84x109/l. Blood film: No blasts seen. Reticulocytes 0.4%. Bone marrow is
hypocellular. What is the most likely diagnosis?
A. aplastic anaemia
B. multiple myeloma
C. agranulocytosis
D. chronic lymphatic leukaemia

294. Suddenly enlarging painful lymph nodes in a patient with chronic myeloid leukemia
indicate:
1. Lymphogranulomatosis
2. Lymph node tuberculosis
3. Viral infection
4. Transformation into acute leukemia

295. A 58-year-old woman complains of fatigability, sleepiness, dyspnea when walking fast. The
analysis of the blood is as follows: erythrocytes — 3.5x1012/l, hemoglobin — 9.2 g/1, Colour Index
- 0.6, plenty of microcytes. What anemia is it?

A. Thalassemia.

B. Iron deficiency.

C. Hemolytic.

D. Pernicious.

296. A woman complains of headache, dyspnea during physical activity. For the last 3 years she
has been having extensive menstrual bleedings. The patient has normal body type, the skin is
pale and dry. The analysis of the blood is as follows: hemoglobin — 9.0 g/1, erythrocytes-3.4-
1012/l, Colour Index - 0.7, ESR - 20 mm/h, significant hypochromia of erythrocytes, anisocytosis,
poikilocytosis. What type of anemia is it?

A. Chronic posthemorrhagic

B. Hemolytic

C. Acute posthemorrhagic

D.B12 and folic deficiency

297.Which of the following findings is diagnostic of iron deficiency anemia -


a) Increased TIBC, decreased serumferritin

b) Decreased TIBC, decreasedserumferritin

c) Increased TIBC, increasedserumferritin

d) Decreased TIBC, increasedsermferritin

298.Patient I. 40 years old, 5 years ago was undergone the resection of stomach because of
peptic ulcer complication. The expressed general weakness, shortness of breath, appeared lately.
Blood test: er. 3,1x1012/l, Hb 60 gr/l, colorindex 0,6, leukocytes 4,5x109/l, eosynophiles 2%, stub
3%, segm. neutr. 55%, lymph. 32%, mon. 8%, ESR 5 mm/hour. What laboratory test will help in
clarification of diagnosis?
a. Levelofilirubin

b. levelofglucose

c. levelofserumiron

d. levelofcreatinine

e. levelofamylase

299.A patient, 60yrs old, complains of general weakness, fatigue, parestesia in


limbs. He had resection of stomach 3years ago because of peptic ulcer. Data of
examination: the tongue is of raspberry colour and smooth, the patient is not
stable in Romberg’s position. Data of CBC: erythrocytes-2,5.1012/l, Hb-88g/l , CI-
1,3, macrocytosis. What disease is present?
a. vitamin B12 deficiencyanemia

b. iron-deficiencyanemia

c. hemolyticanemia

d. hypochromicanemia
e. hyperchromicanemia

300.A patient, 60yrs old, complains of general weakness, fatigue, parestesia in limbs. He had
resection of stomach 3years ago because of peptic ulcer. Data of examination: the tongue is of
raspberry colour and smooth, the patient is not stable in Romberg’s position. The patient is
diagnosed vitamin B12 deficiency anemia. How do you explain parestesia?

a. Affection of central nervous system

b. affection of peripheral nervous system

c. dystensionofskin

d. affectionofsubcutaneousfat

e. allmentioned

301.A patient,35yrs old, who suffers of autoimmune gastritis and vitamin B12-deficiency
anemia, presents signs of funicular myelosis. Data of CBC: erythrocyte-2,2.1012/l, Hb-80g/l, LC-
2,4.109/l, ESR-40mm/hr, direct bilirubin-4,6micmol/l, indirect bilirubin-27micmol/l. What is
funicular myelosis?

a. demyelinization of posteriolateral columns of spinal cord

b. demyelinization of anterior columns of spinal cord

c. affectionofbraincortex

d. demyelinization of peripheral nerves of lower limbs

e. nocorrectanswer

302. Chronic myelogenous leukemia is characterized by the following pattern bone marrow:
1. diffuse proliferation by granulocyte cells
2. fatty bone marrow
3. lymphoid proliferation
4. hyperplasia of red sprout

303. In the blood of patient, 36 years, who treated the respiratory viral infection by the
sulfanilamides, there is hyporegenerative normochromic anemia, leukopenia, thrombocytopenia.
In the bone marrow - reducing of myelokariocytes. What is the kind of anemia?

A. Aplastic

B. Hemolytic

C. Posthemorrhagic

D. B12 - deficiency
E. Iron - deficiency

304. A 77-year-old woman is admitted to the hospital because of difficulty walking. She has had
progressive pain and paresthesia of both feet over the past 3 weeks. Her pulse is 80/min,
respirations are 16/min, and blood pressure is 130/80 mm Hg. Neurologic examination shows
decreased ankle jerk reflexes bilaterally and decreased vibratory sense and proprioception in the
lower extremities. Laboratory studies show:Hemoglobin 8 g/dL, Leukocyte count -2,4.109/l,
platelet count 90,0Х109/l. Mean corpuscular volume 106 μm3

Which of the following is the most likely diagnosis?

(A) Diabetic neuropathy

(B) Hypothyroidism

(C) Pulmonary osteoarthropathy

(D) Spinal stenosis

(E) Vitamin B12 (cobalamin) deficiency

305. A 21-year-old woman comes to the physician for counseling prior to conception. She
delivered a female newborn with anencephaly 1 year ago. The newborn died at the age of 4 days.
She asks the physician if she can take any vitamins to decrease her risk for conceiving a fetus
with anencephaly. It is most appropriate for the physician to recommend which of the following
vitamins?

(A) Vitamin B12 (cyanocobalamin

(B) Folic acid

(C) Vitamin B1 (thiamine)

(D) Vitamin B2 (riboflavin)

(E) Vitamin B6 (pyridoxine)

306.A 21 year old male presents with anemia and mild hepatosplenomegaly. His Laboratory
studies show: Hemoglobin 8 g/dL, Color index – 1,0, Reticulocytes- 4%, Leukocyte count -
4,4.109/l, platelet count 200,0 Х109/l. , history of single blood transfusion is present till date.
Most probable diagnosis is-
a) Thalassemiamajor

b) Thalassemiaminor

c) Thalassemiaintermedia

d) Autoimmunehemolyticanemia
307.Which of the following is associated with an ыpectrin deficiency in the RBC membrane?
a) Autoimmunehemolyticanemia

b) Hereditaryspherocytosis

e) Thalassemiamajor

c) Thalassemiaminor

308. Complications of hemorrhagic vasculitis


1. chronic glomerulonephritis
2. urolithiasis
3. cardiovascular failure
4. Depression of hematopoiesis

309.Which ONE of these is the most typical blood count in a patient presenting with chronic
myeloid leukaemia?

a.  Hb 3 g/dL; WBC 10х109/L; platelets 500х109/L

b.  Hb 9 g/dL; WBC 90х109/L; platelets 400х109/L

c.  Hb 6 g/dL; WBC 12х109/L; platelets 50х109/L

d.  Hb 16 g/dL; WBC 10х109/L; platelets 500х109/L

310.Which ONE of these is considered as an optimal response to imatinib treatment for chronic
myeloid leukaemia?

a. blood count that returns to normal after 6 months of treatment

b.  Failure to find any Philadelphia‐positive chromosomes in bone marrow samples after 12


months

c. Failure to detect any mutations within the BCR‐ABL gene after 12 months

d. Noneoftheabove

311.  What is seen in multiple myeloma?

A. Increased Alkaline phosphatase


B. Decreased IgA

C. Hypercalcemia

D. Hypouricemia

E.  Fat bone marrow

312.   Bence Jones proteins are excreted in the urine in

A. Chronic lymphocytic leukemia

B. Waldenstroms macroglobulinemia

C. Rheumatic fever

D. Multiple myeloma

E. AML

313. The main diagnostic criteria of multiple myeloma include

A. Lytic bone marrow lesions

B. Decreased B2 microglobulin

C. Bence Jones proteinuria

D. Plasmacytosis>10%

E. Rouleaux formation on peripheral smear

314. A 63-year-old patient complains of pain in spine, head, fever, fatigue, weakness, weight
loss. Physical and laboratory examinations find out tender bones, normocytic and normochromic
anemia, granulocytopenia and thrombocytopenia. Bone x-ray examination of ribs shows lytic
bone lesions. What is the possible diagnosis?

A. Polycytemiarubravera

B. AML

C. Multiple myeloma.

D. CML

E.  CLL

315. A 64-year-old patient complains of weight loss, night sweating, general weakness, fatigue,
sensation of fullness in left hypochondrium, easy bruising. CBC reveals: RBC 3.4 х 1012/L, Hb
102 g/L, Ht 41 %, MCV 78 fl, ReticulocyteCount 1.0 %, Pl 75 х 109/L, WBC 125,4 х 109/L,
SegmentedNeutrophils 33 %, BandNeutrophils 8 %, Metamyelocytes 3 %, Myelocytes 5 %,
Myeloblasts 23 %, Monocytes 3 %, Eosinophils 6 %, Basophils 18 %, Lymphocytes 1 %, ESR
33 mm/h. Whatisthemostlikelydiagnosis?

A. Acute leukemia
B. Chronic myeloid leukemia, accelerated phase
C. Chronic lymphocytic leukemia

D. Chronic myeloid leukemia, blast crises

E. Multiple myeloma

316 A 32 -year-old patient complains of weakness and fever. His illness started as tonsillitis a month before. On
. exam, BT of 38.9°C, RR of 24/min, HR of 100/min, BP of 100/70 mm Hg, hemorrhages on the legs,
enlargement of the lymph nodes. CBC shows RBC 2.0х 1012/L, Hemoglobin 65 g/L, Hematocrit 41 %,
MCV 84 fl, ReticulocyteCount 1.0 %, Pl 67 х 109/L, WBC 124,2 х 109/L, SegmentedNeutrophils 15 %,
BandNeutrophils 5 %, Blasts 65 %, Monocytes 10 %, Lymphocytes 5 %, ESR 38 mm/h.
Whatisthecauseofanemia?
A. Chronic lympholeukemia

B. Acute leukemia
C. Aplastic anema
D. Vitamin B12 deficiency anemia
E. Chronic hemolytic anemia

317. Following changes were discovered in peripheral blood of patient: erhytrocytes - 3,0x1012/l Hb-80g/l,
leukocytes - 1,0x109/l. Leukocyte formula: basophyles - 0%, eosinophyles - 0%, myeloblasts - 64%, stab - 1%,
segmentonuclear - 8%, lymphocytes - 24%, monocytes- 3%. Make the diagnosis due to blood picture.
A. Chronic myeloid leukemia.
B.
D. Erythremia.
Acute myeloid leukemia

318. For what disease is characterized detection in urine Bence-Jones?

A multiple myeloma.

B. Hodgkin's disease.
B. chronic glomerulonephritis.

G. acute leukemia.

D. autoimmune hemolytic anemia.

319. A 50yo man presents with itching after hot shower with dizziness,

chest pain after exercise. Exam: splenomegaly. Hb-19.2g/dl What is the single most likely
causative factor?
k. ALL
l. Lymphoma
m. Polycythemia
n. Eczema

320. A 51-year-old female with a history of treatment for thyroid dysfunction-carbimazole


therapy, presents with a sore throat, fever 39.00c. Examination reveals swollen and painful
fauces.Hemogramm: leukocytes 0,9x109/l Choose the SINGLE most likely cause
A. pernicious anaemia
B. agranulocytosis
C. haemolyticanaemia
D. paroxysmal nocturnal haemoglobinuria

321. A 17-year-old man presents with fever, neck stiffness and photophobia. On examination he
is hypotensive and peripherally shut down, with purpura over his legs. There is persistent oozing
from his venepuncture sites. His blood film contains many red cell fragments.
Choose the SINGLE most likely cause
A. hereditary spherocytosis
B. sickle cell anaemia
C. b-thalassaemia major
D. disseminated intravascular coagulation

322.Coagulopatyconsumptionat DIC-syndromeischaracterizedbysuchphenomena:

A. Hypercoagulation
B. Infarctions
C. Lysisofbloodmicrothrombus
D. Hypocoagulation

323.The initialphaseof DIC-syndromeischaracterizedby:


a. Thedystrophicchangesinorgans
b. Hypocoagulation
c. Formationofstablefibrin
d. Hypercoagulation

324. Hemorrhagic syndrome in aplastic anemia


is due to:
1. abnormal structure of the wall vessels
2. increased destruction of platelets in the blood
3. Deficiency of coagulation factors
4. increased platelet consumption
5. Inadequate formation of megakaryocytes

325. Your actions in anemic coma in a patient with B12 deficiency anemia:
1. Whole blood transfusion
2. Transfusion of red blood cells + vitamin B12
3. hydrocortisone in / in, polyglucin
4. Prednisolone IV, transfusion of red blood cells

326. The method of treatment of hereditary microspherocytosis is:


1. Anabolic steroids
2. corticosteroids
3. vitamin therapy
4. Splenectomy

327. A 42-year-old patient complains of back pain, dark urine, general weakness, dizziness occurred
after treating a flu with aspirin and ampicillin. Physical examination: the patient is pale, with
subectericsclerae. HR - 98 bpm. Liver - +2 cm, spleen - +3 cm. Peripheral blood smear: RBCs -
2,6 x 1012/l, Hb - 60 g/l, CI - 0,9, WBCs - 9,4 x 109/l, basophils - 0,5%, eosinophils - 3%, band
neutrophils - 6%, segmented neutrophils - 58%, lymphocytes - 25%, monocytes - 7%, ESR - 38
mm/hour, reticulocytes - 24%. Total bilirubin - 38 millimole/l. What complication occurred in
A. the patient?
Acquired hemolytic anemia
B. Toxic hepatitis
C. Cholelithiasis
D. Agranulocytosis
E. Paroxysmal nocturnal hemoglobinuria

328. A 37-year-old woman sees her physician because of gum bleeding, menorrhagia. Physical
examination reveals petechiae, bruises on her legs. CBC: RBC 2.5 х 1012/L, Hemoglobin 67
g/L, MCV 64 fl, CI – 0,7, Pl 41 х 109/L, Giantplatelets +, WBС 7.9 х 109/L,
SegmentedNeutrophils 60%, BandNeutrophils 4%, Monocytes 5 %, Eosinophils 3 %, Basophils
2 %, Lymphocytes 27 %, ESR 7 mm/h, lactatedehydrogenase (LDH), indirectbilirubinlevels,
bloodureanitrogen (BUN) andcreatininemeasurementsare  normal. Bleeding time 13 min. What
is the most probably diagnosis?
A. Idiopathic thrombocytopenic purpura
*
B. Henoch–Schonlein purpura
C. Haemolytic uremic syndrome
D. Chronic myelogenous leukemia
E. Macrocytic hyperchromic anemia

329.The following laboratory determinants is abnormally prolonged in ITP -

a) APTT b) Prothrombintime

a) Bleeding time d) Clotting time

330.Which one of the following platelet counts is usually associated with increased incidence of
spontaneous bleeding - )
a) Greatertha 80,000/mm3 b) 40,000/mm3

c) 20,000mm3 d) Less than 20,000mm3

331. The typical clinical sighs of Schönlein-Henoch purpura are:

(A) Anemia and jaundice, splenomegaly;

(B) Acute onset with appearance of multiple ecchymoses, petechiae on the lips


and buccal mucosa, epistaxis

(C)Intermittent fevers; bone pain; pallor, petechiae, and purpura; hepatomegaly or splenomegaly;


generalized lymphadenopathy.

(D)    Purpuric skin rash, migratory polyarthritis or polyarthralgia, colicky abdominal pain,


nephritis.

(E)  Hemarthroses, massive bleeding after traumas, operations, and teeth extraction.

332. The boy, 4 years old, is hospitalized in clinic with nasal bleeding, which had appeared in 1
hour after trauma. From anamhesis: often hemarthroses, from 1 year of life. In the general blood
analysis: er. 3.1х1012/l, Hb 89 г/л, thrombocytes 165х10 9/l, duration of the bleeding is 2
minutes, coagulation time is 15 minutes. What disease is possible in this child?

(А) Werlgof's disease 

(B) Hemophilia

(C) Hemorrhagic vasculitis

(D) Willebrand's disease 

(E) DIC-syndrome

333. The girl, 7 years old, is hospitalized in clinic with complains of acute abdominal pain,
sickness, vomiting, hemorrhagic rashes on skin. Hemorrhagic vasculitis is suspected.  What
therapy is necessary to prescribe this child?

(А) Anticoagulants

(B) Spleenectomy

(C) Cytostatics

(D) E-aminocapronic acid

(E) Cryoplasm

334.Which of the following is the best source of factor VIII -a) Fresh foodb) Fresh frozen plasma
c) Cryoprecipitate d) Platelet concentrate

335. Increased bleeding due to impaired primary hemostasis is characteristic for:


1. Hemophilia
2. Secondary coagulopathy
3. Thrombocytopenic purpura
4. Willebrand disease
5. congenital coagulopathy

336. Which of the laboratory indicators is characteristic for hemophilia?


1. Prolonged bleeding time
2. Decrease in the number of platelets
3. Prolongation of clotting time
4. Impaired platelet function
5. Decrease in the number of megakaryocytes

337. At what disease in patients are observed intermuscular, subcutaneous, retroperitoneal


hematomas?
1. Hemorrhagic vasculitis
2. Thrombocytopenic purpura
3. Hemophilia
4. Willebrand disease

338. Characteristic for hemophilia A laboratory test:


1. Deficiency of the VIII coagulation factor
2. Coagulation factor II deficiency
3. Deficiency IX factor of coagulation
4. Deficiency of the XII factor of coagulation

339. At what disease plasmapheresis is one of the components of the basic therapy?
1. Hemorrhagic vasculitis
2. Thrombocytopenic purpura
3. Hemophilia
4. Rundu-Osler disease

340. Why is fresh frozen plasma used for hemorrhagic vasculitis?


1. For replenishment of coagulation factors
2. For stimulation of immunity
3. To improve microcirculation
4. For activation of fibrinolysis
5. For replenishment of antithrombin III

341 Which of following is seen in haemophilia A


.A.
menorrhagia,

B. easy bruising (purpura),

C. extravasation of blood from capillaries into skin


D. hematomas
E. extravasation of blood from capillaries into mucous membranes (petechiae).

342. A 7-year-old girl presents with a 3 day history of rash and ankle swelling. She had a cold 4
weeks previously. On examination she has palpable non-blanching purple spots 1-4 mm in
diameter especially over thelower extremities and buttocks. Her left ankle is swollen, warm and
tender, with restricted movement.
What is the most likely diagnosis?
A. Acute lymphoblastic leukaemia
B. Henoch Schonlein purpura
C. Idiopathic Thrombocytopaenic Purpura
D. Pneumococcal septicaemia

343. Patients with immune thrombocytopenic purpura (ITP) have


A. Normal-to-increased number of megakaryocytes

B. Elevated blasts

C. Elevated lymphocytes

D. Elevated plasmocytes

E. Decreased precursors of RBC

344.A 73-year-old woman present with bruising on extremely minor trauma and widespread
purpura. Her platelet count is 14x10 9\ L and a bone marrow shows megakaryocytes suggestive
of immune thrombocytopenia purpura.
Choose the SINGLE most appropriate treatment.
A. Desmopressin
B. Recombinant factor VIII concentrate
C. Heparin
D. Prednisolone
E. Warfarin

345.A 4-year-old child presents to ER with spontaneous bleeding into both knee joints. He has
had a number of admissions relating to bleeds previously. Lab. test: A normal PT and platelet
count but prolonged APTT
What is the most likely diagnosis?
A. haemophilia
B. multiple myeloma
C. agranulocytosis
D. chronic lymphatic leukaemia

346.A patient with an Hb of 6 g%, WBC count of 2000/cmm, has a normal different count
except for having 6% blasts, platelets are reduced to 80,000/cmm; moderate splenomegaly
is present. Possible diagnosis is:
A. Leukemia
B. Aplastic anemia
C. HEMOLYSIS
D. lTP

347.A 3-year-old girl presents with pallor and marked gland enlargement. She has been unwell
for the past 3 weeks. On examination the temperature is 37.6°C, and she looks pale. She has a
few petechiae on the neck and palate, with moderate generalised lymphadenopathy and an 3 cm
spleen. What is the most likely diagnosis?
A. aplastic anaemia
B. multiple myeloma
C. Folic acid deficiency
D. acute leukaemia

348.A 20 year old university student comes to the student health center because of marked
fatigue. Temperature is 38.3°C (101.0°F). Physical examination shows striking pallor of skin,
nail beds and conjunctivae. There are petechial hemorrhages in the skin of his legs. A soft,
blowing systolic murmur is present over the precordium. No other abnormalities are present. The
most appropriate study at this time is:
a) Complete blood count
b) Determination of bleeding and clotting time
c) Examination of bone marrow aspirate
d) Hemoglobin electrophoresis
e) Serological testing for infectious mononucleosis
349.A 23-year-old male with Down syndrome is brought to your office by his parents. The
patient has had a fever for 12 days. The fever is not associated with rhinorrhea, sore throat,
cough. The parents note that their son has complained of aches in various places, and has not
been eating well for several weeks. The physical examination is remarkable painless lumps in the
neck, pale conjunctivae, petechiae on lower extremities and an 5 kg weight loss since his last
visit 1 month ago.

Which one of the following is most consistent with this patient’s symptoms?
a) Acute myeloid leukemia
b) Atypical pneumonia
c) Hodgkin’s lymphoma
d) Myocarditis
e) Urinary tract infection

350.A 54-year-old woman complains of increasing fatigue and easy bruising of 3 weeks’
duration. Physical findings included pale, scattered ecchymoses and petechiae and mild
hepatosplenomegaly. CBC: RBC – 2.5x1012/L; Hb – 73 g/L; HCT 20%; PLT – 23.000/mcL; and
WBC – 162x109/L with 82% of blasts. Whatisthemostprobablediagnosis?
a. Acuteleukemia

b. Chronicleukemia

c. Thrombocytopenia

d. Hemolyticanemia

e. Megaloblasticanemia

350. A pt. with an Hb of 6, WBC count of 2000, has a normal Different count except for having
6% blasts; platelets are reduced to 80,000; moderate splenomegaly is present; possible diagnosis
is -
a) Leukemia b) Aplasticanemia

c) Hemolysis d)ITP

You might also like